#EMLE November 2024 Exams
#EMLE November 2024 Exams
#EMLE November 2024 Exams
• st
1 Trial
• nd
2 Trial
• st
1 Simulation
• nd
2 Simulation
1. A 40- year- old man who had undergone a vagotomy for duodenal ulcer suffered a
recurrent painful ulcer not responsive to treatment. At an operation for hemorrhage an
inflammatory peripyloric ulcer mass was avoided by performing an antrectomy and Billroth
II (Polya) partial gastrectomy. Soon afterwards he developed a stomal ulcer, confirmed on
endoscopy. What laboratory Investigations should be performed to diagnosis?
A. Blood gases.
B. Serum gastrin.
C. Serum glucagon.
D. Serum Na & K.
E. Serum T3, T 4, TSH.
2. A 37-week newborn is delivered through vaginal delivery. At 1 minute after birth, the
baby was blue, limp, with irregular gasping respiration, heart rate of 60 bpm, and no
response to nasal catheter. What is his most probable APGAR score?
A. 0
B. 2
C. 4
D. 5
E. 6
3. During a training session for medical staff, a lecturer emphasizes the importance of
maintaining patient confidentiality. What does “confidentiality” mean in the context of
patient-doctor relationships?
4. An 80-year-old female presents with confusion associated with a chest infection. She
received standard treatment, and four days later she developed green, then bloody
diarrhea. Sigmoidoscopy showed yellow white plaques adherent to hyperemic mucosa.
Which one of the following organisms is most likely to be responsible for her diarrhea?
A. Campylobacter jejuni
B. Clostridium difficile
C. Escherichia coli
D. Methicillin-resistant Staphylococcus aureus
E. Vancomycin- resistant enterococcus
2 EMLE November 2024 Exams
5. A 2700- g, 36- wk's-GA white male is born after 22 hrs of premature rupture of the
amniotic membranes. The Apgar scores are 3 and 5. He immediately experienced
respiratory distress and cyanosis requiring ET Intubation and MV with 100% O2. Vital signs
are T: 36.2°C, HR 195 bpm, mean BP 22 mm Hg. WBC 1500/ 11 l, platelets 59,000/ 11 L.
Which of the following Is the most appropriate treatment for this baby?
A. Surfactant by aerosol
B. IV ampicillin and gentamicin/cefotaxime
C. IV steroids
D. IV acyclovir
E. High frequency oscillatory ventilation
6. A 60-year-old woman comes to the emergency room in a coma. The patient's temperature
is 32.2°C. She is bradycardic with puffy eyes and face. Her thyroid gland is enlarged. There
is diffuse hyporeflexia. BP is 100/60. Which of the following is the best next step in
management?
7. A 36-year-old female presented with pallor, puffiness of the eye lids and edema of the
lower limbs. Her BP was 180/100. Investigations revealed a 24 hour protein of 8 gm/dl, 2-3
RBCs in urine, serum creatinine of 1mg/dl, her complement levels were normal. ANA and
antiDNA were negative. Which of the following is the likely diagnosis?
A. Nephritic syndrome
B. Heart failure
C. Lupus nephritis
D. Nephrotic syndrome
E. Cluster headache
8. A 60-year-old woman has had a 6-month history of bloody diarrhea. She presents to the
ER with severe abdominal pain and constipation. On examination, her abdomen is
distended and tender on the left side. Rectal examination reveals small amount of feces.
Blood tests show Hb 11.0 g/dl TLC 15.6 x 109/1. platelets 450 x 109/L Sodium is 139 mmol/l
(N: 135-145), potassium is 4.6 mmol/l (N: 3.5-5), urea 25 mg/dl creatinine 1.3 mg/dl and C-
reactive protein (CRP) is 75 mg/l. What is the next best of further investigation?
A. Colonoscopy
B. CT of the abdomen
C. Magnetic resonance imaging (MRI) of abdomen
D. Sigmoidoscopy
E. X-ray of abdomen
EMLE November 2024 Exams 3
9. A 75-year-old man develops bleeding per rectum and presents with blood pressure of
90/60 mmHg and heart rate of 120 beats/mm. His vital signs improved slightly with
crystalloid and packed red cells infusion. Which of the following is considered the
appropriate next step in management?
A. Barium enema
B. Colonoscopy
C. Conservative medical treatment
D. CT scan with double contrast (IV and oral)
E. Radioactive labeled RBCs study
11. A 3-year-old boy with fair hair color and bad smell of urine presented with intellectual
disability, he has an affected older sister with inborn error of metabolism. Which of the
following is a possible diagnosis?
A. Phenylketonuria
B. Galactosemia
C. Mucopolysaccharidosis
D. Gaucher disease
E. Tyrosinemia
12. A 2-year-old girl comes to the clinic with fever, bruising and generalized petechiae all
over her body. Temperature 37.8 C. Two weeks before she had rhinitis and mild cough for 3
days. Examination shows no other physical abnormality. Hemoglobin 12.3 gm/dl, Platelet
count of 21,000/mm3. What the most probable diagnosis?
14. A 6-year-old has a sore throat and has been given antibiotics. Three weeks later, he
represents feeling feverish with nausea, vomiting and tea-colored urine. Urine dipstick
confirms hematuria and protein. Pressure is 100/60mmHg. What is the most likely
diagnosis?
A. Nephritic syndrome
B. UTI
C. Acute tubulointerstitial nephritis
D. Minimal change glomerulonephritis
E. Post streptococcal glomerulonephritis.
15. A 34-year-old, non-lactating Para3, started to use combined oral contraceptive pills
(COCs) after delivery as before. Her last pregnancy was complicated by lower limb deep
venous thrombosis (DVT). Which of the following is the proper advice for contraception?
16. A 67-year-old man is complaining of easy fatigability. A routine complete blood count
(CBC) reveals a platelet count of 800,000/ml, and the hemoglobin and WBC counts are
normal. He reports no other symptoms, and his clinical examination is normal. Which of the
following characteristics is most likely to be helpful in differentiating essential (primary)
from reactive (secondary) thrombocytosis?
A. Mitral stenosis
B. Mitral regurgitation
C. Aortic stenosis
D. Aortic regurgitation
E. Ventricular septal defect
18. You are called to see an 85-year-old female patient as the nursing staff is concerned
that the patient has not passed stool for 4 days. The patient has been admitted after family
members became increasingly concerned regarding her general deterioration in health and
level of function. She is orientated but frail and complains of increasing abdominal
discomfort. On examination bowel sounds are increased. The abdomen is distended with
generalized tenderness, but no rebound or guarding. There is a firm palpable mass in the
left iliac fossa. Digital rectal examination shows an empty rectum. What diagnosis must be
excluded?
A. Simple constipation
B. Paralytic ileus
C. Sigmoid volvulus
D. Peritonitis secondary to diverticular disease
E. Neoplasia
19. An 8-month-old boy presents to the emergency room with jerky movement of all limbs.
He was a full-term baby, with no neonatal problems. He is exclusively breast fed. He is
afebrile with height and weight at 50th percentile. What is the next investigation to be
carried out?
A. Lumbar puncture
B. Electromyography
C. Nerve conduction velocity
D. Serum calcium
E. Serum sodium
20. A 5-year-old male presents with confirmed rotavirus diarrhea. He is tachycardic and
lethargic with sunken eyes, poor skin turgor, and dry mucous membranes. Which of the
following is the most appropriate next step in management?
21. A 12-day-old girl born at 32 weeks becomes lethargic and hypothermic over the last 24
h. She is not tolerating her formula feeds, has 2-episodes of bilious vomitous, and 3-times
bloody diarrhea. On examination: abdominal distention, visible bowel loops, abdominal
wall erythema, and absent bowel sounds. Which of the following is the most likely
diagnosis?
A. Hirschsprung's disease
B. Duodenal atresia
C. Esophageal atresia
D. Necrotizing enterocolitis
E. Meconium ileus
22. A 25-year-old G3 P1+1 presents to the emergency room complaining of lower abdominal
crampy pain 6 weeks from her last normal period. She had significant vaginal bleeding, but
no passage of tissue & pregnancy test is Positive. Which of the following is the most
important step in this patient's evaluation?
A. Sonography
B. Physical exam
C. CBC
D. Quantitative B-hCG
E. Detailed menstrual history
23. A 30-year-old primigravida presents at 34 weeks' gestational age with blood pressure of
170/100 mmHg, headache, epigastric pain, visual abnormalities and 3+ proteinuria.
Biophysical profile of the fetus is 10/10. Which of the following is the immediate step in
management?
A. Start anti-hypertensives
B. Start magnesium sulfate
C. Give betaclomethasone to induce fetal lung maturity.
D. Perform an amniocentesis to assess fetal lung maturity.
E. Repeat the biophysical profile daily.
24. A 25-year-old man developed bilateral loin pain, frank hematuria, and edema in feet. His
symptoms had started 24 hours after developing a sore throat, his blood pressure was
130/80 mmHg. Urine analysis was positive for blood (4+) and protein (2+). Which of the
following is the most likely diagnosis?
A. IgA nephropathy
B. Microscopic polyangiitis
C. Nephrolithiasis
D. Post-streptococcal glomerulonephritis
E. Septicemia
EMLE November 2024 Exams 7
25. A 15-year-old adolescent girl is diagnosed with meningococcemia. She has a 4-year-old
brother and an 11-month-old sister at home. What is the appropriate management of her
siblings?
26. A 27-year-old man was brought to casualty by ambulance following a fall from the third
floor of a tower block. He was conscious and fully oriented. The casualty officer achieved
venous access with two 14-gauge cannula and requested a full blood count and 2 units of
group-specific blood. Cervical spine and chest radiographs were normal; however, the
radiograph of the pelvis showed a clearly displaced fracture through the left pubic rami and
disruption of the left sacroiliac joint. On examination he had a pulse of 120 beats/min and a
blood pressure of 95/70 mmHg. His scrotum was severely bruised and swollen and a small
amount of blood was present at the external penile meatus. Abdominal examination
revealed a suprapubic tender swelling reacting to the level of the umbilicus which of the
following is other injury suggested in association with this fracture?
27. A 38-year-old woman presents to the GP with pain in the subareolar region of the left
breast associated with occasional blood-stained nipple discharge. Apart from being
extremely anxious she has no other associated symptoms. Examination is unremarkable.
What is the most likely diagnosis?
A. Duct ectasia
B. Galactocoele
C. Intraductal papilloma
D. Paget disease
E. Prolactinoma
28. A 60-year-old diabetic man complains of chest pain of 2 hours duration. He is diabetic.
The patient is sweaty, pulse is 110 b/min and Bp is 90/60. There is bilateral basal crepitation
over the back of his chest. ECG showed raised ST segment in V1 till V6. Troponin is
positive. Which of the following is the most likely diagnosis?
A. Stable angina
B. Unstable angina
C. Acute myocardial infarction
D. Acute pulmonary embolism
E. Acute pericarditis
8 EMLE November 2024 Exams
29. A 65-year-old man who underwent an anterior resection for rectal cancer complains of
pain in his left calf on the first postoperative day. On examination he has low-grade pyrexia
(37.5°C), and the calf looks swollen with shiny skin and is tender. The patient experiences
pain on dorsi flexing his foot. What is the appropriate diagnosis?
30. A 16-year-old girl presents with acute onset of maculopapular rash, fever, and oliguria
one week after receiving ampicillin for streptococcal pharyngitis. Her temperature is 38.7 C,
blood pressure is 110/70 mmHg, pulse is 94/min, and respirations 16/mm Urine analysis
shows microscopic hematuria, increased leukocytes with numerous eosinophils, and
occasional white blood cell casts. No proteinuria. Blood tests: elevated antistreptolysin O
titer and moderate eosinophilia. Blood urea nitrogen BUN Is 42 mg/dL, and serum
creatinine is 2 5 mg/dL Which one of the following is the most likely diagnosis?
31. A 65-year-old woman with a past medical history of hypertension and stable angina
presents to the GP practice complaining of episodes of severe central abdominal pain.
They occur around half an hour after meals and can last up to an hour. She is becoming
reluctant to eat because of the pain and as a result she has lost nearly a stone in weight
over the past 2 months. Examination is unremarkable. Which of the following of these
investigations would be the most informative?
32. The parents of an 18-month-old girl bring her to the ER after she had a seizure. They
reported she was in a good health, her temperature 39,5 C. She is oriented and playing
now. Which of the following suggest the good outcome of her condition?
33. A 28-year-old, Para2 used combined oral contraceptive pills for the last 8 months as
advised by one of her friends. Her mother died of breast cancer 3 years ago. What is the
appropriate advice regarding contraception?
34. A 33-year-old man with a history of scaly skin lesions in the elbows and knees
condition. Two years after onset of skin lesions he develops destructive deformity of the
joints in hands, which of the following is the most likely diagnosis?
A. Rheumatoid arthritis
B. Gout
C. Ankylosing spondylitis
D. Arthritis mutilans
E. Osteoarthritis
35. 63-year-old man, with an 8-year history of hepatitis C infection and well-documented
cirrhosis and portal hypertension, presents with a large hematoma on his thigh. On
preoperative screening, his prothrombin time is noted to be 17.4 seconds (N: 11-13.5
seconds). Transfusion of which of the following is the most appropriate next step in
management of this patient prior to his procedure?
A. Cryoprecipitate
B. Fresh frozen plasma
C. Packed red blood cells
D. Platelets
E. Whole blood
36. A 34-year-old primigravida, at 28 weeks, presented at her routine antenatal visit with a
complaint of vaginal discharge diagnosed to be vaginal candidiasis, for which she received
two successive courses of local treatment with incomplete resolution. Her Urine analysis
showed pus cells 50-60/high power field, sugar 1+. On ultrasound examination, the
amniotic fluid index was 23. What is the next step of management?
37. After a year of 4-monthly follow-up, a healthy 75-year-old woman with a 5 cm simple
unilocular ovarian cyst and a normal serum CA-125 level decides that she would prefer to
have surgical treatment, which of the following treatment would you recommend?
38. A 2-year-old boy with the spastic diplegic cerebral palsy is being evaluated. He was
born prematurely and spent 2 weeks in the NICU for respiratory distress management.
What findings can MRI of his brain show?
A. Multicystic encephalomalacia
B. Periventricular leukomalacia
C. Normal anatomy
D. Basal ganglia abnormalities
E. Agenesis at the corpus callosum
39. 26-year-old Para 1+2 came to the emergency department 3 weeks after an abortion, with
severe vaginal bleeding. She reported undergoing surgical evacuation 3 weeks earlier but
did not return to receive the pathology report of the specimen. Her beta- subunit titer was
above 100,000 milli-international units/milliliter, her ultrasound showed an intra-uterine
mass 4x4 centimeters. What is the most appropriate management?
40. A 28-year-old girl presents with oligo-hypomenorrhea, and facial hirsutism. Her physical
examination is normal, and transvaginal ultrasound shows multiple subcortical small cysts
4-6 mm in both ovaries. Which of the following is the most likely laboratory finding?
41. A 60-year-old heavy smoker man presented with hemoptysis for one month. CT showed
bronchogenic carcinoma with liver metastases. The diagnosis was confirmed by
bronchoscopic biopsy. To whom should you explain the situation and prognosis?
A. Brother
B. Daughter
C. Son
D. Patient himself
E. Wife
EMLE November 2024 Exams 11
42. A 7-month-old infant is evaluated for gastrointestinal bleeding and easy bruising.
Physical examination shows shortened forearms, bruising and petechias. Radiograph of
her forearms shows bilateral absent radii. Her CBC is normal with the exception of a
platelet count of 13,000/mm3. What management do you offer to the family?
43. A 52-year-old man has been recently diagnosed with type 2 diabetes and has been
following a plan of lifestyle measures to improve his diet and increase his level of exercise
for the last 3 months. On returning to clinic his BMl is 28, fasting plasma glucose 154
mg/dl, glycated hemoglobin HbA1c 7.5%, s creatinine 0.9 mg/dL, BP 135/80 mmHg. Which
of the following is the most appropriate next management step?
A. Glibenclamide
B. Metformin
C. Insulin
D. Continue diet and exercise regimen only
E. Sleeve gastrectomy
44. A previously fit 59 years old male presented with a small amount of dark red blood and
mucous mixed in the stool. There has been a recent change of bowel habits, but no
significant abdominal or anal pain and plain abdominal radiographs were normal, which of
the following is mostly a cause of this pattern of gastrointestinal bleeding?
A. Diverticular disease.
B. Ulcerative colitis.
C. Hemorrhoids
D. Colonic neoplasia
E. Ischemic colitis
46. A 14-year-old male presents to his GP due to shortness of breath and chest discomfort
whilst exercising. On examination there is a double apical impulse and a harsh mid systolic
murmur loudest between the apex and the left sternal border. His ECG shows left
ventricular hypertrophy (LVH) and widespread Q waves. You suspect Hypertrophic
Obstructive Cardiomyopathy (HOCM), which of the following is associated with increased
risk of sudden death?
47. A 30-year-old woman presents with fever, night sweats, and muscles ache for the past 1
month. She was previously well. On examination, she has axillary and cervical lymph
nodes, but no active joints or hepatosplenomegaly. She currently works in a pig farm. Her
investigations are negative for EBV, CMV, and HIV. Serologic tests for the infecting agent
confirm the diagnosis in her. Which of the following is the most likely diagnosis?
A. Brucellosis
B. Histoplasmosis
C. Leprosy
D. Infectious mononucleosis
E. TB
48. A 56-year-old man with recently diagnoses of chronic obstructive lung disease (COPD)
presents with shortness of breath and wheeze. Blood gases reveal a PO2 of 60 mm Hg
(Normal: 75 to 100 mm Hg), PCO2 of 53 mm Hg (Normal: 35 to 45 mm Hg), PH of 7.25 and a
bicarbonate of 24 mEq/L (Normal: 22 to 26). What do these gases indicate?
49. A female patient, 75-year-old presented with epigastric mass & weight loss with change
in bowel habit towards constipation, the patient passed stool today in the morning.
Colonoscopy was done early and revealed a nearly obstructing mass in the transverse
colon and biopsy showed adenocarcinoma grade III. What is the surgical option?
51. A 3-year-old girl brought to the emergency room for evaluation of fever and cough for 3
days. Her temperature was 39.5 °C, HR 120bpm, RR 30/min, SpO2 96% on room air. On
exam, she is alert with no respiratory distress. Diminished breath sounds are noted in the
right base. Which antibiotic would be the most appropriate for that girl?
A. Trimethoprim sulfamethoxazole
B. Acyclovir
C. Ceftriaxone
D. Azithromycin
E. Amoxicillin
52. A 53-year-old man presents to the GP with a deep, painful ulcer over the big toe. He
gives a 3-month history of severe calf pain on walking which is only eased on resting.
Examination shows cool peripheries with reduced distal pulses. Which ulcer does the
patient most likely have?
A. Arterial ulcer
B. Curling ulcer
C. Marjolin ulcer
D. Neuropathic ulcer
E. Venous ulcer
53. A 66-year-old man had an anterior resection of the rectum five days ago. Now, he has
increased abdominal pain, his temperature is 38.5°C, pulse is 120 bpm, and his blood
pressure is 90/60 mmHg. His abdomen is tender and there is generalized rigidity. What is
the most likely diagnosis?
A. hypovolemia
B. leaking anastomosis
C. secondary hemorrhage
D. urinary tract infection
E. wound infection
14 EMLE November 2024 Exams
54. A patient is in a critical condition and needs immediate surgery to survive however the
patient refuses the surgery based on personal religious beliefs. What legal and ethical
principles should guide the healthcare team in this situation?
55. Which of the following is the most effective method to prevent the transmission of
healthcare associated infections?
56. A 5-year-old girl presented with prolonged fever, progressive pallor, arthralgia, and skin
ecchymosis. Which of the following is the MOST likely diagnosis?
A. Wilms’ tumor
B. Hepatoblastoma
C. Leukemia
D. Rhabdomyosarcoma
E. Brain tumor
57. A 26-year-old female patient presents with increase in the vaginal discharge and
dysuria for two days duration. She is sexually active with her husband and uses condoms
intermittently. Examination reveals some erythema of the cervix. Urine culture is negative.
Sexually transmitted disease testing revealed gonorrheal infection. Which one of the
following infections should be treated concomitantly?
A. Bacterial vaginosis.
B. Chlamydia.
C. Herpes.
D. Syphilis.
E. Trichomoniasis.
58. After 3 months of marriage, a couple (husband 38 years old and wife 31 years old),
Consulted their doctor being anxious about their fertility. Their premarital assessments
were unremarkable. Which of the following is the appropriate management?
59. An 18-year-old male patient had long term history of suppurative otitis media, was
presented to neurology department with high grade fever, headache, vomiting, and
photophobia. On examination, chills, severe prostration and neck stiffness were prominent.
The neurologist started immediate treatment after receiving a CSF sample. CSF profile
showed elevated protein and decreased sugar. What is the causative organism of this
condition?
A. Mycobacteria
B. Fungal
C. Bacterial
D. Protozoal
E. Viral
60. A 31-year-old man is brought to the emergency room following a motor car accident in
which his chest struck the steering wheel. Examination reveals stable vital signs, but the
patient exhibits multiple palpable rib fractures and paradoxical movement of the right side
of the chest. Chest X-ray shows no evidence of pneumo-or hemothorax but a large
pulmonary contusion is developing. What is the proper management for this case?
61. A 3-week-old male infant cries after feeds that last 2-hours. He often calms down after
passing gas. He passes stools after each feed. He receives cows' milk formula His mother
recently noted flocks of blood in the stools. Which of the following is most likely causes his
symptoms?
A. Malrotation
B. Pyloric stenosis
C. Hirschsprung disease
D. Milk protein intolerance
E. Mild ulcerative colitis
62. A 25-year-old woman consulted her gynecologist regarding her primary infertility for
one year. Her husband's semen analysis revealed azoospermia in spite of his satisfactory
hormonal profile. Which of the following is the appropriate next step for the management of
this couple?
A. In-vitro fertilization.
B. Intra-uterine insemination.
C. Testicular biopsy.
D. Post-coital test.
E. Prostate ultrasound
16 EMLE November 2024 Exams
64. A 25-year-old female patient, sustained a laceration of the perineum during delivery, it
involved the muscles of perineal body but not the anal sphincter. Which of the following is
the degree of this laceration?
A. First degree
B. Second degree
C. Third degree
D. Forth degree
E. Fifth degree
65. A 23-year-old man presents with prolonged nasal bleeding. He has always noted easy
bruising, and ongoing bleeding after minor cuts. There is no prior history of surgery or
dental procedures. His hemoglobin is 14.5 g/dL, platelets 200,000/ml, and PT/PPT is normal.
Further testing reveals that the bleeding time is elevated; the factor VIII level is reduced, as
is the ristocetin cofactor assay, which of the following is the most likely diagnosis?
67. A 40-year-old man, 18 hours after a gastrectomy, suddenly becomes hypotensive with a
blood pressure of 80/60, tachycardia of 110/min, CVP +2 cm, H2O and a temperature of
39oC. The likely diagnosis is:
A. Ruptured anastomosis
B. Hemorrhage
C. Acute gastric dilatation
D. Pulmonary embolism
E. Septicemia
EMLE November 2024 Exams 17
68. A 13-year-old boy arrives to the ED 3-hours after feeling a sudden left testicular pain
white playing basketball. Ho had nausea and vomiting. On examination a tender and
swollen left testicle. It is displaced superiorly and lying transversely. There is absent
cremasteric reflex on the left. The right testicle is normal in location and is non-tender.
Which of the following should be the next step in management?
69. A 27-year-old man presents to the GP practice having noticed a painless swelling of his
right testicle. He is otherwise well. On examination, the testis is enlarged, firm and has a
nodular texture. What is the most likely diagnosis?
A. Epididymal cyst
B. Gumma
C. Hematocele
D. Orchitis
E. Testicular cancer
71. A 61-year-old man presents with rapid onset left iliac fossa pain and fever. He is tender
in the left iliac fossa but there is no obvious palpable mass. Temperature is 38.7°C, pulse
94 bpm, white cell count 16 X 109/L, and CRP 60 mg/L. Which of the following is the single
most likely diagnosis?
A. Acute appendicitis
B. Sigmoid cancer
C. Crohn’s disease of the colon
D. Acute sigmoid diverticulitis
E. Ureteric colic
18 EMLE November 2024 Exams
72. A 62-year-old man presents to the GP with a lump in the left groin which has been
present for over 2 months. On examination, the lump is above the inguinal ligament. It is
reducible and has a cough impulse but does not extend into the scrotum. Which of the
following is the most likely diagnosis?
73. A 3-month-old baby with low grade fever, wheezing and dry cough. His older brother
had recent upper respiratory tract infection. On examination: RR 72/min, hypoxia. CXR
shows hyperinflation and some infiltrate. What is the most likely diagnosis?
A. Croup
B. Epiglottitis
C. Bronchial asthma
D. Bronchiolitis
E. Pneumonia
74. A 9-month-old boy brought to the ER with history of watery diarrhea for the past week.
His blood pressure = 90/54, pulse = 102, respiratory rate = 18, and altered Level of
consciousness. Which of the following lab is expected?
A. Metabolic acidosis
B. Respiratory acidosis
C. Metabolic alkalosis
D. Respiratory alkalosis
E. Normal ABG
A. Diabetic ketoacidosis
B. Lactic acidosis.
C. Hyperglycemic hyperosmolar state.
D. Hyperchloremic acidosis
E. Salicylate intoxication
76. A 30-year-old man cut his hand while changing his flat tire yesterday evening. He says
that he had a tetanus booster 4 years ago and full childhood immunization. Which is the
single most appropriate management?
77. A one-year old boy brought to the well-baby clinic to assess his developmental
milestone. Which of the following this infant is age appropriate?
A. Kicks a ball
B. Climbs up and down from furniture without assistance
C. Says short sentences with two to four words
D. Builds towers with four or more blocks
E. Stand unsupported
78. A 27-year-old Primigravida came for her first antenatal visit at 6 weeks gestation. Her
routine antenatal care was unremarkable. Which of the following should be given as
necessary supplement?
A. Iron
B. Calcium.
C. Iron and folic acid.
D. Vitamin C
E. Pregnancy multivitamin formula
79. A 26-year-old lady P2+0 presented with secondary amenorrhea of three years duration.
FSH and LH were found to be high. What is the most likely diagnosis?
A. Sheehan syndrome
B. Asherman syndrome
C. Premature ovarian failure
D. Imperforated hymen
E. Pituitary adenoma
80. A 3-year-old child presented with fever for 6 days not responding to parenteral
antibiotics. Mouth cavity showed significant erythema without ulcers. Cervical lymph
nodes were bilaterally enlarged but larger on the left in addition to conjunctival congestion.
Platelet count was 780,000/mms. What is the most effective line of treatment?
A. Combined IV antibiotics
B. IV acyclovir
C. IV IG
D. Antifungal therapy
E. IV steroid
82. A 25-year-old unmarried female patient, consulted her doctor about her menstrual
pattern during the last five months. Inspite of having regular menses, she has mid-cycle
vaginal spotting and cramping pelvic pain. Her examination was unremarkable, and she
has normal pelvic ultrasound scan as well as a normal vaginal cytology. Which of the
following is the most likely cause of her complaint?
83. A 24-year-old man, who has been suffering from intermittent fresh bleeding per rectum,
presents to the emergency department with a 6-hour history of right-sided abdominal pain,
fevers, and nausea. On examination, he has tenderness and guarding in the right iliac
fossa. His temperature is 38.2°C. What is the most likely diagnosis?
A. Appendicitis
B. Hemorrhoids
C. Meckel diverticulitis
D. Renal colic
E. Shigella infection
84. A 45-year-old male patient, presented to the outpatient clinic with a 3-month history of
epigastric pain associated with epigastric fullness, diarrhea and weight loss. Computed
tomographic (CT) scan showed calcification in the pancreatic head and a 7 cm cystic lesion
in the tail of the pancreas. Which of the following is the most likely diagnosis?
A. Acute pancreatic.
B. Chronic pancreatic.
C. Mucinous cystadenoma of pancreas.
D. Papillary cystadenoma of pancreas
E. Pseudo pancreatic cyst
85. A 39 male patient complains of right iliac fossa pain and swelling right sided tumors of
the large bowel present more frequently with which of the following characteristics when
compared to left sided tumors?
87. A 3-month-old male comes to the emergency room with a history of upper respiratory
symptoms for 3 days, fever, decreased appetite and increased work of breathing. On exam
his vitals are T37.2°C, RR 40, HR 185, BP 90/65, and Sat 95% on room air; he has mild
retractions and coarse breath sounds. Which of the following is the MOST appropriate next
step in management?
88. A 36-year-old Para 5 has just delivered vaginally. After delivery of the placenta, brisk
vaginal bleeding is noticed. The uterus is soft and fails to maintain adequate contraction.
Her pulse rises to 125 beats per minute and blood pressure drops to 80/40 mmHg. Which of
the following the FIRST step of management?
89. A 27-year-old man complains at perianal itching bleeding, discharge, pain and lumps for
a few months. On examination he has pinkish-white swelling outside and inside the anal
canal partially obscuring the anal orifice. What is the appropriate diagnosis?
A. anal fissure
B. fistula in-ano
C. hemorrhoids
D. hidradenitis suppurativa
E. sebaceous cyst
22 EMLE November 2024 Exams
A. ERCP stenting
B. pancreaticoduodenectomy
C. Pancreaticoduodenectomy + hepatic resection
D. total pancreatectomy
E. radiotherapy only
91. A 23-year-old woman has a 6-month history of amenorrhea and galactorrhea. Physical
examination confirms that milk can be expressed from both breasts, but it is otherwise
unremarkable. The pelvic examination is also unremarkable. Visual field examination is
normal. A pregnancy test is negative. Magnetic resonance imaging (MRI) showed a pituitary
microadenoma (4 mm in diameter) and serum prolactin 160 ng/ml (normal level up to 25
ng/ml). Which of the following is the most appropriate management?
92. A 79-year-old man with no significant past medical history has become progressively
more forgetful in the past 9 months. He has trouble remembering familiar places and
people can no longer balance her checkbook and has increased trouble verbally
expressing her thoughts. These symptoms have progressively worsened in the past
several months. He has no history of head trauma or of anxiety disorder. His neurologic
examination is within normal limits except for a mini-mental status examination score of 22
of 30, missing points on calculation, recall, and orientation. A rapid plasma reagin antibody
test (to assess allergic reactions) was negative, and the thyroid stimulating hormone,
vitamin B12, folate, and electrolytes were normal. Which of the following is the most likely
diagnosis?
A. Alzheimer's disease
B. Creutzfeldt-Jakob disease
C. Delirium
D. Depression
E. Normal pressure hydrocephalus
EMLE November 2024 Exams 23
93. A 20-year-old female is brought to casualty with a one-day history of not talking to
anyone, not taking food and incontinence. On examination all vital parameters are stable,
but she is entirely unresponsive and responds only to painful stimulus, her eyes are open
and lusterless (dimmed or unable to see clearly). What is the most likely diagnosis?
A. Delirium
B. Malingering
C. Intoxicated with unknown substance
D. Stupor
E. Drowsy
94. In a pediatric ward, a child with chickenpox has been admitted. The nursing staff is
discussing precautions to prevent the spread of the virus. What precaution should be
primarily implemented to limit transmission among other patients and staff?
95. An 18-year-old man presented to casualty with a 3-hours history of acute pain in the
right hemiscrotum. There were no associated symptoms. There was no history of
frequency, dysuria or urethral discharge. The patient gave a history of similar shorter
episodes of right-sided scrotal pain of sudden onset, short duration and rapid resolution in
the preceding 12 months. On examination the patient was afebrile with a soft abdomen.
Examination of the scrotum revealed that the right testicle was lying higher in the scrotum
than its companion and the spermatic cord was noted to feel thicker than the left spermatic
cord. The right testicle and spermatic cord were extremely tender. Which of the following is
the expected outcome from treatment?
96. A 25-year-old woman presents in the emergency department with acute palpitations.
She is fully conscious, pulse 150 bpm, BP 125/75 mmHg, respiratory rate 20/min, oxygen
saturation 99% on air, chest auscultation is clear with no evidence of cardiac failure. ECG
shows narrow complex regular tachycardia. What is the most appropriate initial
management step?
A. DC Cardioversion
B. Intravenous lidocaine
C. Carotid sinus massage.
D. Intravenous adenosine
E. Digitalis
24 EMLE November 2024 Exams
97. During routine follow-up of a 38-year-old lady on oral contraceptives for 7 years, a Pap
smear was done. The result showed presence of cells suggestive of high grade squamous
intra-epithelial lesion (HSIL). Which of the following is the proper further management?
99. A 32-year-old man presented with low back pain for 3 months. The pain is worse on
awakening and gets better on movements. On examination, there was limitation in forward
and lateral bending of the trunk. What is the most useful test for diagnosing the cause of
his back pain?
100. A 33-year-old presents with a one-week history of malaise, fever, headache, myalgia
with a sore throat, dry cough and pleuritic chest pain. On examination there is bilateral
basal crepitations. There is evidence of erythema nodosum and bloods reveal a hemolytic
anemia and cold agglutinins. Given the most likely diagnosis, what treatment should be
commenced?
A. Erythromycin
B. Ciprofloxacin
C. Amoxicillin
D. Co-trimoxazole.
E. Co omoxiclav
November 2024, 30th October
(Second Trial Exam Medicine)
EMLE November 2024 Exams 1
1. A 40-year-old man presents to the emergency department with emesis of bright red
blood. Laboratory results include HGB 10 g/dl, HC 30%, platelets 300,000/mm3,
international normalized ratio (INR) 1.0, aspartate transaminase (AS) 30 U/L, alanine
transaminase (AL) 45 U/L, and albumin 4.0 g/dl. After appropriate resuscitation, he
undergoes esophagogastroduodenoscopy (EGD) which is notable for gastric varices.
Which of the following is the appropriate treatment or his condition?
2. An 80-year-old man with history of symptomatic cholelithiasis presents with signs and
symptoms of a small-bowel obstruction. Which of the following findings would provide the
most help in ascertaining the diagnosis?
4. An infant present to the emergency room with bilious emesis and irritability, Physical
examination is notable for abdominal tenderness and erythema of the abdominal wall.
Abdominal X-ray demonstrates dilated proximal bowel with air-fluid levels. Which of the
following is the most appropriate next step in management, after resuscitation?
A. Colonoscopy
B. Upper gastrointestinal series
C. Barium enema
D. Gastrostomy
E. Laparotomy
2 EMLE November 2024 Exams
5. A man who was involved in a high-speed automobile collision arrives at the emergency
department in a deep coma. His pupils react poorly to light but are of equal size. An airway
placed, and the patient is sent for CT scan of the head with extension to the neck. The
study shows no cervical spine fractures, but does reveal a small, crescent-shaped
hematoma on the night hemisphere with no deviation of the midline structures. Which of
the following is the room appropriate next step in management?
A. High-dose steroids
B. Hyperventilation, diuretics, and fluid restriction
C. Systemic vasodilators and alpha blockers
D. Surgical evacuation of his epidural hematoma
E. Surgical evacuation of his subdural hematoma
6. A 22-year-old man is stabbed in the right chest with a 5-cm-long knife blade. On arrival at
the emergency department, he is awake and alert. He is speaking with a normal tone of
voice but complaining of shortness of breath. The right hemithorax is hyper-resonant to
percussion and has no breath sounds. The rest of the initial survey is negative. His blood
pressure is 110/75 mm Hg pulse is 86 beats/min, and venous pressure is 3 cm H2O. Pulse
oximetry shows a saturation of sex. Which of the following is the most appropriate next
step in patient care?
A. Intra-uterine adhesions
B. Endometrial polyp.
C. Endometriotic spots
D. Chronic endometritis
E. Decreased ovarian reserve.
8. A 54-year-old man presents to the ED with acute onset of severe abdominal pain. His
history is significant for gnawing epigastric pain that radiates to the back for several
months. Physical examination demonstrates mild hypertension and tachycardia as well as
a rigid board like" abdomen with generalized rebound tenderness and hypoactive bowel
sounds. Rectal examination reveals dark hemoccult positive stools without gross blood.
Which of the following would be the next appropriate step in management?
9. You are a second-year surgery resident and have just left work after a 30-hour shift. On
your way home, you witness a recent collision where there is an obviously injured man.
Several bystanders are providing care for the injured victim. You elect to keep driving a
witness at the scene recognizes you as a physician and reports you to the authorities for
neglecting to stop to provide care. As a consequence of your actions. Which of the
following will most likely happen?
10. A 50-year-old woman was diagnosed with perimenopausal bleeding. During her DSC
biopsy for that bleeding, the gynecologist suspected uterine perforation by the uterine
sound. Which of the following is the most appropriate next step in patient care?
11. 45-year-old man visits his gastroenterologist complaining of chronic abdominal pain,
bloating, and alternating diarrhea and constipation for the past two years. He mentions that
his symptoms often wake him up at night and are not relieved by passing stool. He reports
no significant weight loss or blood in his stool. On physical examination, there are no-
signs of abdominal tenderness or masses. Which of the following additional features would
be most indicative of irritable bowel syndrome in this case?
A. Rectal bleeding
B. Nausea
C. Weight loss
D. Nocturnal symptoms
E. Positive fecal occult blood test
12. A 72-year-old man has a 4-cm hard mass in the left supraclavicular area. The mass is
movable and non-tender and has been present and steadily growing for the past 3 months.
The patient had a vague feeling of epigastric discomfort over the past 2 months. Physical
examination shows evidence of the weight loss. The supraclavicular mass is obvious, but
no other masses can be felt anywhere else in the neck, axillae, or groins. There is occult
blood in the stool, and his hemoglobin is 10.5 g/dl. Which of the following would a biopsy
of the supraclavicular mass most likely reveal?
A. Chronic inflammation
B. Lymphoma
C. Metastatic gastric cancer
D. Metastatic squamous cell carcinoma
E. Metastatic thyroid cancer
4 EMLE November 2024 Exams
13. A 4-year-old girl was brought to the emergency room for evaluation of fever and cough
for 3 days. Her temperature was 39.5 °C, HR 120 beats/minute, RR 30/minute, SpO2 96% on
room all. On examination, he is alert with no respiratory distress. Diminished breath
sounds are noted in the right base. Which of the following actions would be appropriate?
14. A 22-year-old man has a temperature of 39.5°C one hour after initiation of a transfusion
of packed red blood cells for a hemoglobin concentration of 7 g/dl Yesterday, he underwent
open reduction and internal fixation of a fractured femur sustained in a motor vehicle
collision. He received prophylactic cefazolin prior to the operation and is now receiving
enoxaparin He is otherwise healthy and has no known allergies. The remainder of the vital
signs are within normal limits. Examination shows no abnormalities. Which of the following
is the most appropriate next step in management?
15. A 10-year-old girl is brought to the causality with difficult breathing and severe
retractions. She is the known asthmatic patient. Which of the following medications is
needed for treatment of this acute asthma exacerbation?
A. Inhaled corticosteroids
B. IV ketamine
C. IV magnesium
D. Nebulized albuterol or salbutamol
E. Nebulized saline
16. Following an uneventful appendectomy for acute appendicitis, the pathology report
reveals the presence of a 1 cm carcinoid at the tip of the appendix. The patient has been
otherwise asymptomatic. Which of the following is the most appropriate intervention?
17. A 35-year-old woman is 33 weeks pregnant in her first pregnancy comes to emergency
department with headache and blurred vision. Her blood pressure is 180/110 mmHg.
Urinalysis shows +++ protein. One week prior, her blood pressure was 120/70 mmHg and
she had no proteinuria. Which of the following is the most appropriate pharmacotherapy?
A. Oral methyldopa
B. Intravenous diazepam
C. Intravenous furosemide
D. Intramuscular betamethasone
E. Intravenous magnesium sulphate
18. An 8-year-old girl is brought to the ER due to sleepiness over the last 2 days.
Temperature is 37.5 °C. Her examination reveals nuchal rigidity. Her CSF analysis: Cells 85
lymphocytes, Protein 150mg (normal 15-45mg%), CSF Glucose 15mg% and serum glucose
130mg%, negative CSF Gram stain Her brain CT with contrast shows enhancement of basal
cisterns. Which of the following is the most likely diagnosis?
A. Tuberous sclerosis
B. Tuberculous meningitis
C. Acute hemorrhagic stroke
D. Acute bacterial meningitis
E. Pseudotumor cerebri
19. An 82-year-old woman comes to her primary care physician's office complaining of
feeling dizzy and light-headed, especially when she stands up quickly. She has a history of
hypertension, diabetes, and osteoarthritis. She also mentions that she has been taking
multiple medications for her various health conditions. Which of the following is the most
likely contributing factor to her symptoms?
20. An 8-year-old girl was brought to the clinic complaining of sore throat. She was
prescribed an antibiotic. Three weeks later, she developed fever and tea-colored urine.
Which of the following is the finding in urine analysis that suggests post-streptococcal
glomerulonephritis?
A. Small volume
B. High specific gravity
C. Red cells in excess
D. Red cell casts
E. Proteinuria
6 EMLE November 2024 Exams
22. A neonate does not pass any meconium during the first day of life. On day 2, he is
brought for the evaluation because of repeated green vomiting and progressive abdominal
distention. X-ray films of the abdomen show multiple dilated loops of small bowel and no
gas in the colon. A contrast enema shows a normally positioned microcolon, and the
contrast material refluxes freely into the small bowel, filling some of the more distal
distended loops. Exploratory laparotomy is done. There is no malrotation, the small bowel
does not have any atretic or obstructed segments, and there is no inspissated meconium in
it. Which of the following is most appropriate next step in management?
A. Diverting ileostomy
B. Diverting ileostomy and appendectomy
C. Transverse loop colostomy
D. Total colectomy
E. Total proctocolectomy and permanent ileostomy
23. A 20-year-old, who is at 14 weeks' gestation, has a 2-day history of vaginal bleeding and
lower abdominal pain. Ultrasound shows a 25 mm fetal pole with absent fetal heart rate,
Pelvic examination reveals her cervix to be 3 cm dilated. Which one of the following is the
most likely diagnosis?
24. A 19-year-old gang member is shot in the abdomen with a 38-caliber revolver. The entry
wound is in the epigastrium, to the left of the midline. The bullet is lodged in the psoas
muscle on the right. He is hemodynamically stable, and the abdomen is moderately tender.
Which of the long the most appropriate next step in diagnosis?
25. A 65-year-old man presents to the clinic complaining of increasing shortness of breath,
especially when lying flat, and swelling in his legs. He also reports feeling fatigued and has
noticed a decrease in his ability to exercise. On further questioning, he mentions waking up
at night with a feeling of suffocation. Physical examination reveals elevated jugular venous
pressure, bilateral pitting edema in the lower extremities, and crackles in the lung bases.
Which of the following symptoms is most indicative of heart failure?
27. A 2-hour-old full-term neonate is noted by the nurses to have episodes of cyanosis.
When they try to feed him, his oxygen level drops to 60%. But when he is stimulated and
cries, his oxygen levels increase above 90%. Which of the following should be done to
diagnose this case?
28. A 56-year-old woman came to the clinic because of chest pain (usually worse with
breathing) anxiety, palpitations (heart racing) and coughing up blood. She was diagnosed
as acute pulmonary embolism. Which of the following is the most frequent ECG finding in
this case?
A. Q wave
B. P. pulmonale
C. Sinus tachycardia
D. Sinus bradycardia
E. Left axis deviation
8 EMLE November 2024 Exams
29. A 32-year-old pregnant woman of 26 weeks of gestation presents to the hospital with
minimal painless bleeding following sexual intercourse. She hemodynamically stable and
has had no such episodes in the post which of the following confirms the diagnosis of
placenta previa?
30. A 32-year-old, para 2 lactating woman complains of increased urinary urgency over the
last week. On her way to the bathroom, most of her urine involuntarily escapes. A urine
culture is negative. Which of the following is the most appropriate next step in her
management?
A. Starting V antibiotics
B. Intravenous pyelography
C. Cystoscopy
D. Fixing on indwelling catheter for one week
E. Urodynamic studies
31. A 12-months old boy is brought to the well-baby clinic, his parents are worried about
his development. Which one of the following is abnormal neurodevelopment?
32. A 34-year-old woman primigravida was known to be a type I diabetic. During her vaginal
delivery, a shoulder dystocia was diagnosed and managed by overdue traction,
exaggerated lithotomy and supra-pubic pressure. Which of the following is the most likely
complication of her baby?
33. A 28-year-old woman is 12 weeks pregnant with a singleton pregnancy. This is her first
pregnancy. Her booking blood pressure, at 12 weeks, is 140/90 mmHg. Her BMI is 34 kg/m2.
She smokes 10 cigarettes/day. Her mother suffered from pre-eclampsia in her pregnancies.
Which one of the following is considered a high-risk factor for the development of pre-
eclampsia as her pregnancy progresses?
34. A 56-year-old woman comes to the clinic complaining of something coming out of her
vagina. She says constipation is a significant problem and that sometimes she needs to
push stool out of her rectum by inserting a finger in the vagina and pressing on a bulge.
Which of the following is the most likely diagnosis?
A. Cystocele
B. Enterocele
C. Rectocele
D. Hemorrhoids
E. Uterine prolapse
35. A 45-year-old man comes to the clinic complaining of fever, chills, and a productive
cough with rust-colored sputum. He has a history of diabetes. He reports feeling generally
unwell for the pass week. On examination, dullness to percussion and bronchial breath
sounds are noted over the right lower lung field. A chest X-ray reveals consolidation in the
right lower lobe. Which of the following is the most likely causative organism for his
respiratory infection?
A. Streptococcus pneumoniae
B. Hemophilus influenza
C. Mycobacterium tuberculosis
D. Pseudomonas aeruginosa
E. Staphylococcus aureus
36. A 12-year-old girl is seen by a pediatrician for a mild case of pneumonia. She is treated
with an intramuscular injection of penicillin. About 15 minutes later, she develops extreme
itchiness, accompanied by the development of wheals scattered over her chest and
extremities. She also begins to wheeze and complains of difficulty breathing. The color of
her lips and face remains rosy. Which of the following is the most appropriate first step in
management?
A. Epinephrine injection
B. IV corticosteroids
C. Intubation
D. No specific therapy is needed
E. Oral corticosteroids
10 EMLE November 2024 Exams
37. A 30-year-old primigravida comes for her booking visit. Her sister had deep vein
thrombosis in her legs last year and suffered much pain and discomfort. She has heard
that pregnancy increases risk for venous thrombosis and wants you to address her
concern. Which of the following is the most appropriate next step in patient care?
A. Warfarin
B. Test for thrombophilias
C. Low-dose aspirin
D. Heparin
E. Reassure
38. A 48-year-old man is admitted to the coronary care unit with an acute inferior
myocardial infarction. Two hours after admission, his blood pressure is 86/52 mmhg, his
heart rate is 40 beats per minute with sinus rhythm. He gives a history of recurrent attacks
of chest pain in the left side of the chest over the last 2 years that always comes after
severe effort. The pain is relieved by rest or some sublingual drugs prescribed by his
doctor. Which of the following would be the most appropriate initial therapy?
39. A 32-year-old woman comes to the clinic complaining of fatigue, nausea, and jaundice
for the past week with dark urine and light stools. She mentions recent travel to a region
known for poor sanitation practices. Physical examination reveals scleral icterus and
hepatomegaly. Laboratory tests show elevated liver enzymes (ALT, AST) and bilirubin
levels. Hepatitis A virus IgM antibodies are detected in her serum. Which of the following is
the meaning of the presence of IgM antibodies against hepatitis A?
A. Acute hepatitis
B. Chronic hepatitis
C. Past infection
D. Carrier state
E. Recurrent
40. In an emergency room, a 45-year-old obese man arrived in intense pain, clutching ha
right abdomen. He was rapidly diagnosed as having acute cholecystitis. He received
immediate treatment with fluids and pain medication. An urgent surgical consult followed
which of the following is the most sensitive image modality for this case?
A. Abdominal CT
B. Abdominal MRI
C. Abdominal ultrasound
D. Abdominal radiography
E. Scintigraphy
EMLE November 2024 Exams 11
A. Bell palsy
B. Erb palsy
C. Klumpka paralysis
D. Pseudobulbar palsy
E. Spinal cord injury
42. A 15-year-old girl presents with a 5-day history of sore throat, low grade fever, and easy
fatigability. Physical examination shows bilateral tonsillar enlargement with exudate. Her
spleen is palpable 3 cm below the left costal margin. Her throat culture is negative for
group A Streptococcus. Monospot test is positive. Which of the following is the most
appropriate management for this patient?
A. Abdominal ultrasound
B. Avoidance of all contact sports
C. Complete blood count
D. Splenectomy
E. Oral penicillin
A. Neisseria gonorrhea.
B. Human papilloma virus.
C. Treponema pallidum
D. Human immune-deficiency virus.
E. Chlamydia trachomatis
44. A 45-year-old woman comes to the clinic complaining of joint pain, swelling, and
stiffness in her hands and wrists for the past three months. On examination, there is
symmetrical swelling of the proximal interphalangeal and metacarpophalangeal joints, as
well as limited range of motion. Laboratory tests reveal elevated erythrocyte sedimentation
rate and C-reactive protein. Radiographs show erosions and joint space narrowing. Which
of the following is the most likely diagnosis?
A. Osteoarthritis
B. Gout
C. Rheumatoid arthritis
D. Systemic lupus erythematosus
E. Psoriatic arthritis
12 EMLE November 2024 Exams
45. A 45-year-old man who complained of heartburn and dyspepsia had received a trial of
proton pump inhibitors. One month later, the symptoms had not improved despite and
findings on endoscopy were unremarkable. Which of the following is the next step in
management?
46. A 33-year-old woman had evacuation for a missed miscarriage 1 week ago, following an
unplanned pregnancy. The histology results suggest a molar pregnancy. She is keen on
contraception to avoid a further unplanned pregnancy. Her beta hCG levels are 960 mlU/I
today. Which of the following contraceptives are best suited to her?
A. Copper IUCD
B. Combined oral contraception
C. Barrier contraception
D. Minipill
E. Mirena IUS
47. A 3-month-old infant is found to have a continuous murmur all over the precordium
during her routine well baby checkup. The infant is 6 kg in weight and appears well. All
peripheral pulses are present and easily palpable. Oxygen saturation is 96%. What is the
most likely finding on echocardiography?
48. A 7-year-old girl has a 5-month history of increasing difficulty in walking associated
with urinary incontinence. Physical examination shows an alert child with exaggerated
deep tendon reflexes, clonus, and bilateral Babinski reflexes in the lower extremities with
grade 3/5 weakness symmetrically in all extremities. Which of the following is the most
likely diagnosis?
A. Guillain-Barré syndrome
B. Medulloblastoma.
C. Myasthenia gravis
D. Compressive myelopathy
E. Polymyositis
EMLE November 2024 Exams 13
49. A 60-year-old man complains of anal itching and discomfort, particularly toward the end
of the day. He works as a salesman in a department store, where he has to be on his feet all
day. When he goes home in the evening, he finds himself sitting sideways to avoid the
discomfort. He has no fever, rectal bleeding, or soiling of his underwear, and he has never
had surgery in that area. Which of the following is the most likely diagnosis?
A. Anal fissure
B. External hemorrhoids
C. Fistula in ano
D. Internal hemorrhoids
E. Perirectal abscess
50. A 2-year-old boy is brought to the clinic with progressive pallor over the last 4 months.
The mother noticed that her baby eats anything even clay. He is on breast feeding till this
age. On examination, he is pale, apathetic looking. No lymphadenopathy, no organomegaly,
and no skin bleeds. Which of the following is the most likely diagnosis?
51. A 55-year-old man with a history of alcohol abuse comes to the clinic because of
abdominal distension, jaundice, and pedal edema. Laboratory tests reveal low serum
albumin, elevated bilirubin, and prolonged prothrombin time. Ultrasound shows signs of
cirrhosis. Which of the following is the most appropriate initial treatment for ascites in this
patient?
A. Oral diuretics
B. Paracentesis
C. Liver transplantation
D. Beta-blockers
E. Antibiotics
52. A 2-year-old girl brought to the emergency department. She is responding to pain but
bradycardic and has poor perfusion. Which of the following is the MOST important first
step in treatment?
53. A 45-year-old man presents with hematuria and proteinuria. Renal biopsy reveals
mesangial deposits of IgA. Which of the following is the most likely diagnosis?
54. A 34-year-old man complained of fever, abdominal pain, diarrhea, and toxic confusional
state He had no symptoms of urinary or respiratory tract disorders. Examination showed
temp: 39 C pulse of 90 B/M and coated tongue with mildly enlarged spleen. Which of the
following is the most likely diagnosis?
A. Brucellosis
B. Food poisoning
C. Botulism
D. Typhoid
E. IBD
55. A 77-year-old man came to the clinic complaining of blood in stool. Rectal examination
revealed a mobile mass. On workup, he was found to have a stage I (Dukes A) well-
differentiated adenocarcinoma. Which of the following is the most appropriate
intervention?
A. Transanal excision
B. Abdominal perineal resection
C. Low anterior resection
D. Placement of endorectal walistent
E. Neoadjuvant chemotherapy followed by Transanal resection
56. A 40-year-old woman comes to the clinic complaining of fatigue, nausea, and decreased
urine output over the past week. She has a history of systemic lupus erythematosus. She
reports tasking nonsteroidal anti-inflammatory drugs for joint pain related to her SLE. On
examination, he appears pale, blood pressure is 150/90 mmHg, heart rate is 100-bpm, and
she has mild pedal edema. Laboratory tests reveal serum creatinine of 3.8 mg/dl and blood
urea nitrogen of-45 mg/dl (normal range: 7-20 mg/dl). Urinalysis shows red blood cell casts
and proteinuria. The patient is diagnosed with acute renal failure. Which of the following is
the most likely cause of the patient's acute renal failure?
A. Glomerulonephritis
B. Acute interstitial nephritis
C. Renal artery thrombosis
D. Acute tubular necrosis
E. Renal vein thrombosis
EMLE November 2024 Exams 15
57. A 3-year-old boy presents to the physician's office with an asymptomatic neck mass
located in the midline, just above the level of the thyroid cartilage. The mass moves with
deglutition and on protrusion of the tongue. Which of the following is/are the most suitable
investigation/s for the case?
A. Aspiration cytology.
B. CBC and ESR.
C. Blood culture.
D. Neck ultrasound
E. Neck CT
58. A multiple trauma patient receives 14 units of packed red cells and several liters of
Ringer's lactate solution during a laparotomy for multiple intra-abdominal injuries. The
surgeons note that blood is oozing from all dissected raw surfaces, as well as from his IV-
line sites. His temperature is normal. Which of the following is the most appropriate next
step in management?
59. A 9-year-old boy is brought to the clinic because of left knee pain. His medical history is
remarkable for hemophilia A. Examination of his left knee shows warmth and swelling there
is pain with flexion. Range of motion is limited by pain and swelling. Which of the following
is the most appropriate therapy for this patient?
A. ADH (vasopressin)
B. Factor VIII concentrate
C. Factor IX concentrate
D. Blood transfusion
E. Fresh frozen plasma
60. A 24-year-old woman presents to emergency department with a 12-hour history of right-
sided chest pain and shortness of breath. She is at 7 days' postnatal having delivered her
baby by emergency caesarean section at 34 weeks. Her pregnancy was complicated by
severe hypertension and postpartum hemorrhage of 1 L. She has a BMI of 32. Her BP is
130/80 mmhg pulse is 108 bpm, temperature is 37.2°C and oxygen saturations are 94% in
air: Which of the following is the most likely diagnosis?
A. Anemia
B. Myocardial infarction
C. Pneumonia
D. Subphrenic abscess
E. Pulmonary embolism
16 EMLE November 2024 Exams
61. A 59-year-old woman undergoes a laparotomy for a suspected ovarian cancer. Her
family asks if you would disclose the results to them so they can explain better to her.
Which of the following ethical principles should guide you to take your decision?
A. Autonomy
B. Beneficence
C. Paternalism
D. Confidentiality
E. Fidelity
62. A 45-year-old man comes to the clinic complaining of abdominal pain, particularly in the
upper abdomen. He describes the pain as a burning sensation that worsens when he hasn't
eaten for a while and improves after eating. He also reports occasional nausea and
bloating. On further examination, the patient reveals a history of smoking and regular use
of NSAIDs for back pain. Which of the following is the most likely diagnosis?
A. Crohn's disease
B. Irritable bowel syndrome
C. Peptic ulcer disease
D. Ulcerative colitis
E. Gastroesophageal reflux disease
63. A 2-year-old girl was brought to the emergency department with sudden severe
hematemesis. History revealed umbilical sepsis during neonatal period. The child appears
pale and physical examination reveals a firm spleen palpable 7cm below the costal margin.
She does mot exhibit hepatomegaly, ascites, or lymphadenopathy, which of the following is
the most likely diagnosis?
A. Migraine headache
B. Cluster headache
C. Subarachnoid hemorrhage
D. Acute angle-closure glaucoma
E. Temporal arteritis
EMLE November 2024 Exams 17
65. Dr. Mohamed works as a general practitioner in a small town where everyone knows
each other. One day, a teenage patient, Mona, comes to see Dr. Mohamed for a confidential
consultation regarding a potential sexually transmitted infection. Mona requests that Dr.
Mohamed keeps this information strictly confidential as she does not want her parents to
find out. However, Dr Mohamed is aware that Mona's parents are close friends with his own
family, and he worries about the potential consequences of withholding such information.
Which of the following should he do?
A. Respects Mona's confidentiality and reassures her that her information will not be
shared without her content
B. Explains to Mona the potential risks of keeping her condition confidential and
encourages her to describe the information to her parents.
C. Seeks guidance from a colleague or supervisor on how to handle the situation while
monitoring patient confidentiality
D. Informs Mona's parents about her condition, citing potential risks to her health if the
information is kept secret
E. Contacts Mona's school counselor or another trusted adult to intervene and ensures
she receives appropriate care without breaching confidentiality
66. A 54-year-old otherwise healthy woman presents to the emergency department with
abdominal pain, ever, chills, and confusion. Blood pressure is 95/50, heart rate 110, and
temperature 39C. Laboratory tests demonstrate a white blood cell count of 15,000, normal
hematocrit and platelets, as well as a direct bilirubin of 7.2mg. Which of the following is the
initial management of this case?
67. A 16-month-old girl is brought to medical attention because of irritability, poor feeding
and temperatures up to 39.4 °C. Careful history and physical examination fall to disclose
any identifiable cause of her fever. There is some degree of abdominal tenderness on
palpation. Which of the following is the most appropriate next step in diagnosis?
A. Abdominal radiographs
B. Culture of urine obtained by transurethral catheterization (clean catch midstream
urine)
C. Microscopic examination and culture of stool
D. Renal ultrasound
E. Voiding cystourethrogram
18 EMLE November 2024 Exams
68. A 28-year-old previously healthy woman presents with persistent cough, fatigue, and
unintentional weight loss over the past months. She denies any recent travel history but
reports close contact with her roommate who was diagnosed with active TB two months
ago. On examination, there are no significant findings except for mild lymphadenopathy.
Chest X-ray shows bilateral upper lobe infiltrates. A sputum sample is collected for testing.
Which of the following is the most appropriate next step in managing this patient?
69. An otherwise healthy 28-year-old man comes to his physician because of painless
enlargement of the right testis. He began to feel a sensational heaviness in the right
hemiscrotum approximately 6 months ago. Physical examination reveals diffuse
enlargement of the right testis, but it is difficult to determine whether this is due to an
intratesticular or extratesticutar lesion. Which of the following is the most appropriate next
step in diagnosis?
A. CT scanning
B. Serum levels of hCG, alpha-fetoprotein, and LDH
C. Scrotal ultrasonography
D. Needle biopsy
E. Inguinal orchiectomy
70. A 22- year- old woman comes to the Gynecology clinic for advice regarding the most
appropriate postnatal contraception. She had an uncomplicated vaginal delivery 6 weeks
back at 40 weeks gestation. She is breastfeeding. She and her partner are keen to space
out childbearing by 2- 3 years and request for the most reliable form of contraception. She
admits to having difficulty in remembering to take contraceptive medication. Which of the
following is the most effective contraceptive option for her?
A. Lactational amenorrhea
B. Progestogen only implant
C. Condoms
D. Combined oral contraceptive pill
E. Combined contraceptive vaginal ring
71. A 23-year-old primigravida come for her first antenatal visit at 8 weeks gestation. She
was clinically free but complaining of some morning sickness. Which of the following is the
most appropriate initial step in patient care?
72. A 35-year-old woman comes to her primary care physician complaining of fatigue,
weakness and shortness of breath upon exertion for the past few months. She reports
heavy menstrual bleeding for the last year but has not sought medical attention until now.
Physical examination reveals pallor of the skin and conjunctiva. Laboratory tests show
microcytic, hypochromic red blood cells with a low hemoglobin and hematocrit, along with
low serum ferritin levels. The patient is diagnosed with iron deficiency anemia and she is
prescribed iron supplements along with further evaluation to determine the cause of her
menorrhagia. Which of the following is used to assess the response to treatment?
73. Six hours after undergoing laparoscopic bilateral inguinal hernia repairs, a 62-year-old
man complains of suprapubic discomfort and fullness. He feels the need to void but has
not been able to do so since the operation. There is a palpable suprapubic mass that is dull
to percussion Palpation of that mass exacerbates the symptoms. Which of the following is
the most appropriate next step in management?
74. A nurse is working in a hospital's intensive care unit where a patient has been
diagnosed with methicillin-resistant Staphylococcus aureus infection. Which of the
following is the most appropriate infection control measure to prevent the spread of MRSA
in the ICU?
76. A 45-year-old woman presents with fatigue, weight gain, and cold intolerance. On
examination, she has dry skin and bradycardia. Laboratory tests reveal elevated thyroid-
stimulating hormone levels with decreased free thyroxine levels. Which of the following is
the investigation of choice for confirming primary hypothyroidism?
A. Thyroid ultrasound
B. Thyroid scintigraphy
C. Thyroid peroxidase antibodies assay
D. Free triiodothyronine measurement
E. Serum thyroglobulin measurement
77. A 55-year-old man presents to the clinic with elevated blood pressure readings. His
medical history includes type 2 diabetes and hyperlipidemia. Which of the following
medications is the most appropriate initial choice for treating his hypertension?
A. Atenolol
B. Lisinopril
C. Amlodipine
D. Hydrochlorothiazide
E. Losartan
78. A 37-year-old woman has been scheduled for an elective caesarean section. She is not
diabetic and of average BMI. Doctors are discussing the operation steps in order to reduce
wound infection. Which of the following should be recommended?
79. A-5 years old boy brought to ER by his mother complaining of drooling saliva, inability
eat and drink. On examination there was fever, congested pharynx and inspiratory stridor.
What is the most appropriate diagnosis?
A. Viral pneumonia
B. Croup
C. Acute epiglottitis
D. Bacterial pneumonia
E. Bronchiolitis
80. A 45-year-old man with a recent diagnosis of purpura seeks advice on activities to
prevent excessive bleeding. Which of the following activities should he avoid to minimize
the risk of bleeding?
A. Swimming
B. Cycling
C. Contact sports
D. Yoga
E. Walking
EMLE November 2024 Exams 21
81. A 45-year-old man was brought to the emergency department with a pale, pulseless,
paresthetic, painful, and paralytic right lower extremity. The process began suddenly 2
hours ago. On examination, no pulses are apparent in the right lower extremity pulse at the
wrist is 95 beats/min and grossly irregular. Treatment would likely be based on which of the
following?
82. A 2-year-old boy was brought to the ER with a one-day history of tachypnea and
respiratory distress. His blood pH was 7.26, PCO2 55 mmhg and bicarbonate of 24mmol/L
were found. Which of the following is the interpretation of this data?
A. Metabolic acidosis
B. Metabolic alkalosis
C. Acute respiratory acidosis
D. Chronic respiratory acidosis
E. Respiratory alkalosis
83. A 55-year-old man with a history of type 2 diabetes mellitus (T2DM) presents for a
routine follow up. His current treatment regimen includes metformin and lifestyle
modifications. His latest HbA1C is 8.5%. Which of the following is the most appropriate
next step in managing his diabetes?
84. A 3-year-old boy was brought to the ER in Coma His Glasgow coma scale was 6. He was
playing at home unattended, he became sleepy 3 hours ago. His examination revealed a
pinpointed pupils. Which of the following is the most likely diagnosis?
A. Barbiturates intake
B. Hypothermia
C. Uncal herniation
D. Severe hypoxic
E. Post-ictal
22 EMLE November 2024 Exams
85. A 25-year-old woman who delivered vaginally two days ago, presents to emergency
department with fever of 38.5 °C, no abdominal pain, minimal vaginal bleeding, no dysuria,
ne sore throat of cough, no lower limb edema or tenderness. She states that the baby was
admitted to the neonatal ICU. Which of the following is the most likely diagnosis?
86. A two-year-old boy is rushed by his mother to the Emergency Room with difficult
breathing and coughing for the last 2 days. Temperature is 37° C, Respiratory Rate 34/min,
HR 100/min. Examination reveals diminished air entry and wheezes on the right side of the
chest only. Chest X-ray reveals over distended right lung, especially in expiratory film.
What is the most appropriate next management step?
87. A 10-year-old boy is brought to the clinic with a 16-day history of fever between 38.3°C
and 38.9°C, and lethargy. Physical examination reveals marked cervical and inguinal
adenopathy, enlarged tonsils with exudates, small hemorrhages on the soft palate,
palpable spleen 2cm below the left costal margin. His WBC lymphocytosis (10% atypical),
Which of the following is the most likely diagnosis?
A. Infectious mononucleosis.
B. Streptococcal throat infection.
C. Tuberculosis.
D. Varicella.
E. Kawasaki disease.
88. A 24- year- old primigravida came to emergency department at 9 weeks with mild
vaginal spotting one day after effort. Ultrasound scan revealed a viable 9 week fetus with a
small caudal separation of the sac. Which of the following is the most appropriate next step
in patient care?
89. A 4-year-old child presented with short breath and edema of both feet. Liver span was
16 cm and hepatic tenderness was noticed. Heart sounds were weak without any murmurs.
X ray of the chest showed significant cardiomegaly. There was history of upper respiratory
catarrh 8 weeks ago. What is the most likely diagnosis?
90. A 32-year-old woman presents with a severe headache, nausea, and sensitivity to light.
She has a history of similar episodes triggered by stress and lack of sleep. Physical
examination is unremarkable. The diagnosis is migraine without aura. Which of the
following is the character of headache of migraine?
A. Stitching
B. Aching
C. Throbbing
D. Burning
E. Constricting
91. A 38-year-old woman sustained a third degree burn in the lateral aspect of her lower leg
when she was 14. The burn was untreated. Since that time, she has had shallow ulcerations
of the scar site that heal and break down all the time. In the past few months, she has
developed an indolent dirty-looking, deeper ulcer at the site, with "heaped up" tissue
growth around the edges. The ulcer is steadily growing and showing no signs of healing.
Which of the following is the most appropriate next step in diagnosis?
A. Doppler studies
B. Venous pressure tracings
C. Culture of the ulcer base
D. Biopsy of the ulcer edge
E. Arteriogram
92. A 62-year-old woman has an eczematoid lesion in the areola of her right breast that has
been present for 3 months. She has self-medicated with skin lotions and over-the counter
steroid ointments, but the area has not improved. On physical examination, the nipple is
inverted, the skin of the areola is reddish and desquamated, and the entire area feels firm,
with no discrete mass demarcated from the rest of the breast. Which of the following is the
most appropriate next step in management?
93. A 39-year-old woman comes to the physician because of a persistent vaginal itch,
vaginal discharge, and dysuria. She has had these same symptoms several times over the
past 2 years and each time has been diagnosed with Candida vulvovaginitis. On physical
examination, she has a thick, white vaginal discharge and significant vulvar and vaginal
erythema. Which off the following is the most appropriate next step in management?
94. A 64-year-old woman complains of right calf pain and swelling. She recently underwent
an uncomplicated left hemicolectomy for diverticular disease. A duplex ultrasound
confirms the presence of deep vein thrombosis (DVT) of the calf. Which of the following is
the most appropriate initial management of this patient?
95. A male newborn infant 2-day-old was diagnosed with Down syndrome, Polyhydramnios
was reported during pregnancy. He has had several episodes of bilious vomiting after
breast feeding. Which of the following would be the most likely finding?
96. A 25-year-old man has multiple intra-abdominal injuries after a gunshot wound
laparotomy reveals multiple injuries to small and large bowel and major bleeding from the
liver. After repos the bowel injuries, the abdomen is closed with towel clips, leaving a large
pack the neve Within 12 hours, there is massive abdominal swelling with edema fluid, and
intra-abdominal pressure exceeds 35 mm Hg. Which of the following is the immediate step
in managing this problem?
97. An 82-year-old man complains of severe abdominal distention, nausea, vomiting, and
colicky abdominal pain. He has not passed any gas or stools for the past 12 hours. His vital
signs are normal, and his pulse is regular, Abdominal X-ray films show distended loops of
snail and large bowel, as well as a very large round gas shadow that is located in the right
upper quadrant and tapers toward the left lower quadrant in the shape of a parrot's beak
The patient has never had any abdominal surgery, and he does not have any palpable
hernias. Which of the following is the most appropriate next step in management?
A. Administering albuterol
B. Administering racemic epinephrine
C. Administering corticosteroids
D. Administering IV penicillin
E. Endotracheal intubation
99. On the second postoperative day after an abdominoperineal resection for cancer of the
rectum a 72-year-old man complains of severe retrosternal pain. The pain is crushing in
nature and radiates to the left arm. He also becomes short of breath and tachycardia.
Except for his fresh surgical wounds and postoperative discomfort, physical examination
is unremarkable. He does not have distended neck veins. Which of the following is the
most appropriate next step in diagnosis?
A. Blood gases
B. Chest x-ray film
C. Pulmonary angiogram
D. Transaminase levels (ALT, AST)
E. Troponins
100. A healthy breast-fed term infant presents to the NICU with jaundice at 36 hours of age.
The total serum bilirubin was 20 mg/dl (0.3-12 mg/dL), direct bilirubin 0.9 mg/dl (0-0.3
mg/dL). Blood film show reticulocytic count of 10% (0.5-2.5%). The baby’s blood group is A
negative and mother’s blood group O negative. What is the most likely diagnosis?
A. ABO incompatibility
B. Breast milk-associated jaundice
C. Criggler Najjar syndrome
D. Hereditary spherocytosis
E. Rhesus hemolytic disease
November 2024, 3rd November
(First Simulation Exam
Medicine)
EMLE November 2024 Exams 1
1. A 52-year-old man has been recently diagnosed with type 2 diabetes and has been
following a plan of lifestyle measures to improve his diet and increase his level of exercise
for the last 3 months. On returning to clinic, his BMI is 28, fasting plasma glucose is 154
mg/dL, glycated hemoglobin is HbA1c 7.5%, s creatinine is 0.9 mg/dL, and BP is 135/80
mmHg. Which of the following is the most appropriate next management step?
A. Glibenclamide
B. Metformin
C. Insulin
D. Continue diet and exercise regimen only
E. Sleeve gastrectomy
2. A 10-year-old boy was brought to the emergency department with high-grade fever,
headache, vomiting, and photophobia. He had a 1-year history of suppurative otitis media.
On examination, chills, severe prostration and neck stiffness were prominent. He started
immediate treatment after a CSF sample withdrawn. CSF profile showed elevated protein
and decreased sugar. Which of the following is the most common causative organism of
this condition?
A. Bacteria
B. Fungus
C. Protozoa
D. Mycobacteria
E. Virus
3. A 9-month-old boy is brought to the ER with a history of watery diarrhea for the past
week. His Blood pressure = 90/54, Pulse = 102, Respiratory rate = 18. He is moderately
dehydrated and has altered Level of consciousness. Which of the following lab findings is
expected?
A. Metabolic acidosis
B. Respiratory acidosis
C. Metabolic alkalosis
D. Respiratory alkalosis
E. Normal ABG
5. A 15-year-old boy was brought to the chest clinic due to recurring episodes of breathing
difficulty accompanied by wheezing sounds, which often occurred after exposure to dust.
His younger sister has a similar condition. Clinical examination revealed scattered rhonchi
throughout the chest. Which of the following drugs is most likely to worsen his condition?
A. Barium enema
B. Colonoscopy
C. Gastrografin follow through
D. Upper gastrointestinal endoscopy
E. Exploratory surgery
7. A 47-year-old male presented to the ER by acute abdominal pain & distension, vomiting,
and constipation for 4 days. The patient gives history of surgery 5 years ago for excision of
abdominal mass which was of benign nature on histopathological examination. Also, there
is history of similar manifestations occurred twice 1 & 3 years after operation. X-ray, U/S,
and CT were done which does not detect pathology except dilated bowel loops. What is the
etiology of these symptoms?
8. 75-year-old patient is found to be acutely short of breath after receiving her third unit of
packed red cells. On examination, there are fine end inspiratory crackles up to mid zones in
both lung fields. What is the most probable diagnosis?
A. Air embolus
B. Delayed hemolytic transfusion reaction
C. Hypocalcemia
D. Viral infection
E. Circulatory overload
EMLE November 2024 Exams 3
11. 4-year-old boy with spastic diplegic cerebral palsy is being evaluated. He was born
prematurely at 32 weeks gestation and spent 2 weeks in the NICU for respiratory distress
management. Which of the following is the most likely to be shown in his brain MRI?
A. Multicystic encephalomalacia
B. Periventricular leukomalacia
C. Normal anatomy
D. Basal ganglia abnormalities
E. Agenesis of the corpus callosum
12. A single 25-year-old female patient presented with galactorrhea. Prolactin level was
high. What is the diagnostic investigation?
A. Breast US
B. Mammography
C. Sonomammography
D. CT brain
E. Cytology
4 EMLE November 2024 Exams
13. A 70-year-old woman with Parkinson’s disease is brought to the Emergency Department
with a 2-day history of rapidly increasing, painless abdominal distension. She has not
opened her bowels in 4 days and has not passed flatus for 2 days. She denies vomiting. On
examination, the abdomen is soft and non-tender but, grossly distended, with a tympanic
percussion note. An abdominal X-ray shows a dilated gas-filled large bowel with a coffee
bean appearance. What is the most likely diagnosis?
A. Constipation
B. Rectal Cancer
C. Femoral hernia
D. Sigmoid Volvulus
E. Pseudo obstruction
14. 45-year-old man is brought to the emergency department with oliguria, vomiting, and
lower limb swelling. On examination, he is earthy looking with bilateral lower limb edema.
Laboratory investigations reveal serum creatinine level 6.7 mg/dl, potassium level 5.4
meq/l, and PH is 6.8. Which of the following is an indication for dialysis in this patient?
A. Hypokalemia
B. Severe metabolic acidosis
C. Hyperuricemia
D. Hypotension
E. Anemia
15. A 65-year-old man presents with sudden-onset severe epigastric pain. He is a smoker
and admits to regular excessive alcohol intake. On examination, he is guarding in the
epigastrium. An erect chest X-ray shows a pneumoperitoneum. What is the most likely
diagnosis?
16. A 35-year-old woman came to the clinic because of palpitation and loss of weight. On
examination, she had exophthalmos and thyroid swelling. Which of the following is the
best initial step for the diagnosis of this patient?
A. RAIU scan
B. Free T4 level
C. Free T3 level
D. TSH level
E. TSI including antithyroglobulin and antimicrosomal Ab
EMLE November 2024 Exams 5
17. A 45-year-old patient presents with acute abdominal pain. Upon examination, the
abdomen is rigid, and there is tachycardia. Laboratory results show leukocytosis, blood
pressure at 90/60 mmHg, blood sugar at 55 mg/dL, sodium at 119 mEq/dL, and potassium
at 6.3 mEq/dL. What is the definitive treatment for this patient?
A. Hypertonic saline
B. 10% dextrose infusion
C. Laparotomy
D. Corticosteroids
E. Vasopressors
18. A 3-month-old boy has been experiencing noisy breathing, particularly during periods
of activity, since he was 2 weeks old. He is feeding well, but upon physical examination, he
presents with loud inspiratory stridor accompanied by moderate suprasternal and
subcostal retractions. Notably, the stridor diminishes when the infant's neck is extended or
when he is placed in a prone position. Both expiration and crying appear normal. What is
the most likely cause of these symptoms?
A. Laryngeal web
B. Tracheomalacia
C. Laryngomalacia
D. Vascular ring
E. Vocal cord paralysis
19. A 4-month-old boy brought to the emergency room with a history of upper respiratory
symptoms for 3 days, fever, decreased appetite and increased work of breathing. On exam
his vitals are T 37.2°C, RR 40, HR 185, BP 90/65, and Sat 96% on room air. He has mild
retractions, and coarse breath sounds. Which of the following is the MOST appropriate next
step in management?
20. A 37-year-old woman has been scheduled for an elective caesarean section. She is not
diabetic and of average BMI. Doctors are discussing the operation steps in order to reduce
wound infection. Which of the following should be recommended?
21. A 65-year-old man comes to the clinic with dyspnea and fatigue. He was previously
diagnosed with ischemic heart disease and atrial fibrillation and was on amiodarone. On
examination, he has clubbing and cyanosis. Which of the following is a side effect of
amiodarone in this patient?
A. Constipation
B. Orthostatic hypotension
C. Pulmonary fibrosis
D. Thrombocytopenia
E. Pneumonia
22. A 56-year-old woman comes to the clinic complaining of something coming out of her
vagina. She says constipation is a significant problem and that sometimes she needs to
push stool out of her rectum by inserting a finger in the vagina and pressing on a bulge.
Which of the following is the most likely diagnosis?
A. Cystocele
B. Enterocele
C. Rectocele
D. Hemorrhoids
E. Uterine prolapse
23. A 32-year-old woman visits the clinic with symptoms of irritability, nervousness, heat
intolerance, and eye protrusion. On examination, her blood pressure is 140/90 mmHg, pulse
is 100/min, and she has an enlarged thyroid gland. After investigations, she is diagnosed
with Graves' disease. What findings would be expected in her hormonal assay?
24. A 48-year-old man has intermittent left-sided lower abdominal pain and progressive
constipation and feels generally unwell. He has lost his appetite and lost weight. His
temperature is 37.3°C, his blood pressure is 150/100 mmHg, and his pulse is 80 bpm. Which
of the following is the best investigation leading to diagnosis?
A. Colonoscopy
B. Fasting serum glucose level
C. MRI abdomen
D. Transrectal ultrasound
E. Ultrasound abdomen
EMLE November 2024 Exams 7
25. A 55-year-old man presents to the Emergency Department with a 2-day history of
vomiting and severe epigastric pain radiating through to his back. He admits to a large
alcohol binge the night before the onset of the pain. On examination, he is tachycardic with
generalized abdominal tenderness and localized guarding in the epigastrium. An erect
chest X-ray shows no obvious abnormality. You suspect a diagnosis of acute pancreatitis.
The surgical registrar asks you to ‘score’ the patient. Which of the following forms part of
the Glasgow Coma Score for acute pancreatitis?
A. Heart rate
B. Systolic blood pressure
C. Temperature
D. PaO2
E. Amylase
26. A 33-year-old woman had evacuation for a missed miscarriage 1 week ago, following an
unplanned pregnancy. The histology results suggest a molar pregnancy. She is keen on
contraception to avoid a further unplanned pregnancy. Her beta hCG levels are 960 mIU/l
today. Which of the following contraceptives are best suited to her?
A. Copper IUCD
B. Combined oral contraception
C. Barrier contraception
D. Minipill
E. Mirena IUS
27. A 42-year-old man comes to the clinic complaining of one day inability to close his left
eye and deviation of the angle of the mouth while speaking. The patient is diabetic on
metformin. On examination: Temperature 37°C, Pulse 75/m, BP 140/85. Chest and heart
examination are normal. Patient is unable to wrinkle forehead on the left side, unable to
close the left eye and unable to blow the cheek. Angle of the mouth deviates to right on
attempt to show the teeth. There is no sensory deficit over the face. Examination of the
tongue shows no abnormality. Which of the following is the neurological diagnosis in this
patient?
A. Ischemic stroke
B. Left facial nerve herpes zoster
C. Left-sided Bell's palsy
D. Left-sided cerebellopontine angle lesion
E. Left-sided trigeminal nerve palsy
28. A 60-year-old heavy smoker man presented with hemoptysis for one month. CT showed
bronchogenic carcinoma with liver metastases. The diagnosis was confirmed by
bronchoscopic biopsy. To whom should you explain the situation and prognosis?
A. Brother
B. Daughter
C. Son
D. Patient himself
E. Wife
8 EMLE November 2024 Exams
29. A 28-year-old woman is 12 weeks pregnant with a singleton pregnancy. This is her first
pregnancy. Her booking blood pressure, at 12 weeks, is 140/90 mmHg. Her BMI is 34 kg/m².
She smokes 10 cigarettes/day. Her mother suffered from pre-eclampsia in her pregnancies.
Which one of the following is considered a significant (high) risk factor for the development
of pre-eclampsia as her pregnancy progresses?
30. A 74-year-old woman presents with a 3-month history of a painful ulcer on the dorsum
of her foot. The ulcer is punched out, has a deep, pale, minimally sloughy base, and is
surrounded by tight, thin dry skin. From the description, what is the most likely etiology?
A. Venous ulcer
B. Arterial ulcer
C. Neuropathic ulcer
D. Diabetic ulcer
E. Malignant ulcer
31. A 32-year-old nulligravida presents to the subfertility clinic with unexplained primary
infertility of 4 years duration which is associated with dysmenorrhea and deep
dyspareunia. She was booked for diagnostic pelvic laparoscopy. Which of the following is
the most likely laparoscopic finding?
A. Intra-uterine adhesions
B. Endometrial polyp
C. Endometriotic spots
D. Chronic endometritis
E. Decreased ovarian reserve
32. A normally fit and healthy 68-year-old man presented to the Emergency Department
after a sudden collapse in the street with sudden onset back pain. On examination, he was
hypotensive and a palpable pulsatile expansile epigastric mass was found, and a diagnosis
of ruptured abdominal aortic aneurysm was made. What is the treatment for ruptured AAA?
A. Do nothing
B. High flow oxygen
C. Open surgical repair
D. IV opioid analgesia
E. IV fluids
EMLE November 2024 Exams 9
33. A 4-year-old girl brought to the clinic with fever, bruising and generalized petechiae all
over her body. Temperature 37.8°C. Two weeks before, she had rhinitis and mild cough for
3 days. Examination shows no other physical abnormality. Hemoglobin 12.3 gm/dl, Platelet
count of 21,000/mm3. Which of the following is the most probable diagnosis?
A. Acute leukemia
B. Aplastic anemia
C. Immune thrombocytopenic purpura
D. Henoch-Schönlein purpura
E. Von Willebrand disease
34. A female patient, 35 years old presented to the emergency room with frequent loose
stool and passage of small amount of red blood per rectum. She recalled that she has
frequent attacks of diarrhea and central abdominal pain over the last 5 months.
Colonoscopy revealed scattered patchy areas of ulcerations alternated with healthy areas.
Which of the following is the most likely cause of the bleeding?
A. Cancer colon
B. Crohn's disease
C. Esophageal varices
D. Ischemic colitis
E. Ulcerative colitis
35. A 25-year-old woman who delivered vaginally two days ago, presents to emergency
department with fever of 38.5 degrees, no abdominal pain, minimal vaginal bleeding, no
dysuria, no sore throat or cough, no lower limb edema or tenderness. She states that the
baby was admitted to the neonatal ICU. Which of the following is the most likely diagnosis?
37. A 23-year-old primigravida came for her first antenatal visit at 8 weeks gestation. She
was clinically free but complaining of some morning sickness. Which of the following is the
most appropriate initial in patient care?
38. A 35-year-old woman is 33 weeks pregnant in her first pregnancy comes to emergency
department with headache and blurred vision. Her blood pressure is 180/110 mmHg.
Urinalysis shows +++ protein. One week prior, her blood pressure was 120/70 mmHg and
she had no proteinuria. Which of the following is the most appropriate pharmacotherapy?
A. Oral methyldopa
B. Intravenous diazepam
C. Intravenous furosemide
D. Intramuscular betamethasone
E. Intravenous magnesium sulphate
39. A 65-year-old man presenting to your clinic with fever, cough, expectoration of
yellowish sputum and dyspnea. On examination he has bronchial breathing and crepitation
over the right upper lobe. Chest X-ray showed upper lobe consolidation. Which of the
following is the most likely diagnosis?
A. Bronchial asthma
B. Lung fibrosis
C. Pneumonia
D. COPD
E. Pleural effusion
40. A 12-year-old boy arrived to the ED 3-hours after feeling a sudden right testicular pain
while playing football. He had nausea and vomiting. On examination, his right testicle was
tender and swollen, and it is displaced superiorly and lying transversely. There is absent
cremasteric reflex on the right. The left testicle is normal in location and is non-tender.
Which of the following should be the next step in management?
41. A 55-year-old man came to the clinic with lower limb edema and abdominal distension.
He had a past history of blood transfusion during hernia operation several years ago. On
performing abdominal ultrasound, liver cirrhosis, splenomegaly and ascites were found.
Which of the following is the likely cause of her cirrhosis?
A. Hepatitis C virus
B. Hepatitis E virus
C. HIV
D. Hepatitis A
E. Cytomegalovirus CMV
42. A newborn girl developed respiratory distress and tachycardia after birth via vaginal
delivery with vacuum assist. She cried spontaneously but remains acrocyanotic, despite
supplemental oxygen delivered by mask. The girl was hypotonic and moves her extremities
only in response to noxious stimuli. Physical examination reveals an open mouth with a
protruding tongue, upslanting palpebral fissures, low-set ears, and a simian crease across
both palms. Which of the following is the most common congenital heart defect in this
condition?
A. Dextrocardia
B. Patent ductus arteriosus
C. Endocardial cushion defects
D. Supravalvular aortic stenosis
E. Pulmonary stenosis
A. Anemia
B. Myocardial infarction
C. Pneumonia
D. Subphrenic abscess
E. Pulmonary embolism
44. A 68-year-old man is admitted to the hospital for pneumonia. During his stay, he
develops a catheter-associated urinary tract infection (CAUTI). Which of the following
infection control strategies is MOST effective in preventing CAUTIs in hospitalized
patients?
45. A 70-year-old cigarette smoker presents with a right inguinal mass that has been
enlarging and causing discomfort in recent months. He also reports recent difficulties with
urination and nocturia. The swelling, which does not extend into the scrotum, reduces
when he rests. What is the likely diagnosis?
46. A 30-year-old lady comes in labor, she gave birth to newborn girl at 32 weeks’ gestation.
She has had no prenatal care. The baby appears to be very pale and shows severe,
generalized edema. Cord-blood hematocrit is 22%, and cord-blood bilirubin is 7 mg/dL.
Ultrasound examination demonstrates pleural effusions, ascites, cardiomegaly, and
hepatomegaly. Which of the following is the most likely diagnosis?
A. ABO incompatibility
B. Beta thalassemia
C. Congenital spherocytosis
D. Sickle cell anemia
E. Rh incompatibility
47. A 7-year-old girl was brought to the emergency room for evaluation of fever and cough
for 3 days. Her temperature was 39.5 °C, HR 120, RR 30, SpO2 96% on room air. On
examination, she was alert with no respiratory distress. Diminished breath sounds were
noted in the right base. Which of the following antibiotics would be the most appropriate
for that girl?
A. Trimethoprim-sulfamethoxazole
B. Acyclovir
C. Ceftriaxone
D. Azithromycin
E. Amoxicillin
A. Neisseria gonorrhea.
B. Human papillomavirus
C. Treponema pallidum.
D. Human immune-deficiency virus.
E. Chlamydia trachomatis
EMLE November 2024 Exams 13
49. A 65-year-old man has an enterocutaneous fistula originating in the jejunum secondary
to inflammatory bowel disease. Which of the following would be the most appropriate fluid
for replacement of his enteric losses?
A. D5W. Dextrose 5%
B. 3% normal saline
C. Ringer lactate solution
D. 0.9% sodium chloride
E. 6% sodium bicarbonate solution
50. You are assessing a 72-year-old man in your pre-operative clinic ahead of an elective
thyroidectomy. He is currently taking warfarin for atrial fibrillation and inquires whether he
should discontinue it before his surgery. What advice do you provide him?
A. No need to stop
B. Omit morning of surgery
C. Stop 5 days before
D. Stop 10 days before
E. Stop 14 days before
51. A 2-week postmature neonate exhibits severe respiratory distress immediately after
birth. Previously, green-tinged meconium was noted in the amniotic fluid. Which of the
following is the most appropriate next step in management?
A. Chest X-ray
B. Suctioning of the mouth and nasopharynx
C. Oxygen supplementation by face mask
D. Intubation with mechanical ventilation
E. Emergency tracheostomy
53. A 6-year-old girl came to the outpatient clinic with fever for 6 days not responding to
parenteral antibiotics. Mouth cavity showed significant redness without ulcers. Cervical
lymph nodes were bilaterally enlarged but larger on the left in addition to conjunctival
congestion. Platelet count was 780,000/mm3. Which of the following is the most effective
line of treatment?
A. Combined IV antibiotics
B. IV acyclovir
C. Intravenous immunoglobulins
D. Antifungal therapy
E. IV steroid
54. A 32-year-old, para 2 lactating woman complains of increased urinary urgency over the
last week. On her way to the bathroom, most of her urine involuntarily escapes. A urine
culture is negative. Which of the following is the most appropriate next step in her
management?
A. Start IV antibiotics
B. Intravenous pyelography
C. Cystoscopy
D. Fix an indwelling catheter for one week
E. Urodynamic studies
55. A 25-year-old woman comes to the emergency department with acute palpitations. She
is fully conscious, pulse is 180 bpm, BP is 80/50 mmHg, respiratory rate is 20/min, oxygen
saturation is 99% on air. Chest auscultation is clear with no evidence of cardiac failure.
ECG shows narrow complex regular tachycardia. Which of the following is the most
appropriate initial management step?
A. DC Cardioversion
B. Intravenous lidocaine
C. Carotid sinus massage
D. Intravenous adenosine
E. Digoxin
56. An 18-month-old girl is brought to the ER after she had a seizure. Her parents reported
that she was in good health, her temperature 39.5°C. She is oriented and playing now.
Which of the following suggests the good outcome of her condition?
57. A 13-year-old girl presents with a 4-week history of intermittent fever, escalating fatigue,
generalized muscle pain, and swelling in both her knees and ankles. She also exhibits a
fine erythematous rash on her back, along with noticeable swelling in her knees and
ankles; the rest of her physical examination reveals no significant abnormalities. Initial
laboratory tests indicate the following: leukocyte count of 11,400 cells/mm³, hemoglobin
level of 8.8 g/dL, blood urea nitrogen of 4 mg/dL, creatinine of 1.4 mg/dL, glucose at 98
mg/dL, C3 complement at 36 mg/dL (normal >80 mg/dL), antinuclear antibody titer of
1:3200, anti-double-stranded DNA titer of 1:640, and negative antineutrophil cytoplasmic
antibodies. Urinalysis reveals moderate hematuria (50 RBC/hpf) and moderate proteinuria
(400 mg/dL). Given these findings, what is the most likely diagnosis?
A. Wegener granulomatosis
B. Giant cell arteritis
C. Henoch-Schönlein purpura
D. Polyarteritis nodosa
E. Systemic lupus erythematosus
58. A 39-year-old woman comes to the outpatient clinic complaining of pain and stiffness in
her hands and wrists for the past 6 weeks. She is diagnosed with rheumatoid arthritis,
although there is no evidence of erosion on x-ray. Which of the following is the
management of choice at this time?
A. NSAID alone
B. NSAID and corticosteroids
C. Corticosteroids alone
D. Corticosteroids and methotrexate
E. Azathioprine
60. A 20-year-old, who is at 14 weeks’ gestation, has a 2-day history of vaginal bleeding and
lower abdominal pain. Ultrasound shows a 25 mm fetal pole with absent fetal heart rate.
Pelvic examination reveals her cervix to be 3 cm dilated. Which one of the following is the
most likely diagnosis?
61. A 65-year-old diabetic man comes to the ER complaining of recurrent retrosternal chest
pain radiating to the shoulder. The pain increases with exertion and improves with rest. His
blood pressure is 150/90 mmHg. Which of the following is the next step to diagnose the
cause of his symptom?
A. ECG
B. Chest x-ray
C. Pulmonary angiography
D. CT chest
E. Chest Ultrasound
62. An 18-year-old male patient had a long-term history of suppurative otitis media, was
presented to the Neurology department with high-grade fever, headache, vomiting, and
photophobia. On examination, chills, severe prostration, and neck stiffness were
prominent. The neurologist started immediate treatment after receiving a CSF sample. CSF
profile showed elevated protein and decreased sugar. Which of the following is the
causative organism of this condition?
A. Bacterial
B. Fungal
C. Protozoal
D. Mycobacteria
E. Viral
63. An 82-year-old woman presents with a 24-hour history of painful swelling in her left
groin. The patient is unwell and dehydrated with a tachycardia of 110 beats/min. On
examination, the swelling is 3 x 3 cm in size, firm, and tender. It is not reducible. What is
the most likely diagnosis?
A. Inguinal lymphadenopathy
B. Femoral hernia
C. Inguinal hernia
D. Saphena varix
E. Femoral artery aneurysm
64. A 39-year-old woman comes to the physician because of a persistent vaginal itch,
vaginal discharge, and dysuria. She has had these same symptoms several times over the
past 2 years and each time has been diagnosed with Candida vulvovaginitis. On physical
examination, she has a thick, white vaginal discharge and significant vulvar and vaginal
erythema. Which of the following is the most appropriate next step in management?
65. An 89-year-old man has had a large pressure sore on the sacrum for the past 2 months.
Medical history includes type 2 diabetes mellitus and multi-infarct dementia. He is
incontinent of stool and urine. Which of the following factors is the most important
contributor to the development of the pressure ulcer in this patient?
66. A 4-week-old boy cries after feeds that last 2 hours. He often calms down after passing
gas. He passes stools after each feed. He receives cow’s milk formula. His mother recently
noted flecks of blood in the stools. Which of the following is the most likely cause of his
symptoms?
A. Malrotation
B. Pyloric stenosis
C. Hirschsprung’s Disease
D. Milk-protein intolerance
E. Mild ulcerative colitis
67. A 1-month-old boy is seen in a well-baby clinic. The mother states that the baby is
constipated and feeds poorly. On examination, he is jaundiced, exhibits poor muscle tone,
and has a large posterior fontanelle and an umbilical hernia. He has gained only 300 g
since discharge from the normal newborn nursery. Which of the following is the most likely
diagnosis?
A. Alpha-1-antitrypsin deficiency
B. Biliary atresia
C. Congenital hypothyroidism
D. Congenital myasthenia gravis
E. Pyloric stenosis
68. A 19-year-old woman has had increasing malaise for the past 5 months. On
examination, she has a systolic murmur best heard at the end of the left border of the
sternum with congested jugular veins, enlarged liver, and bilateral edema of both lower
limbs. Laboratory investigation: Serum bilirubin: 1.0 mg/dL (N. 0.2 – 1.2 mg/dL) Serum
albumin: 3.6 gm/dL (N. 3.5 to 5.4 g/dL), Serum creatinine is 1.5 mg/dL (N. 0.84 to 1.21).
Which of the following is the most likely cause of edema in this patient?
A. Cardiac edema
B. Respiratory edema
C. Hepatic edema
D. Nutritional edema
E. Renal edema
18 EMLE November 2024 Exams
69. A 30-year-old executive learns that he has a duodenal ulcer. His gastroenterologist
prescribes and outlines medical therapy. The patient worries that if medical therapy fails,
he may need surgery. Which of the following is the best indication for elective surgical
therapy for duodenal ulcer disease?
A. An episode of melena
B. Repeated episodes of pain
C. Pyloric outlet obstruction due to scar formation from an ulcer
D. Frequent recurrences of ulcer disease
E. Referral of pain to the back, suggestive of pancreatic penetration
70. A 54-year-old man presents to the hospital with a 3-day history of severe nausea,
vomiting, and diarrhea. He has been unable to keep down substantial solids or liquids over
the past few days and has become progressively weak. He also endorses subjective fevers
and occasional abdominal pain. The patient’s medical history is significant for
hypertension and chronic low back pain, for which he takes lisinopril and ibuprofen. He has
a 4-year-old child that he picks up from daycare who has also had diarrhea. On
examination, the patient is tachycardic with a blood pressure of 104/84 mmHg. He appears
weak with dry mucous membranes. Routine laboratory values are drawn, which show a
blood urea nitrogen (BUN) and creatinine of 40 mg/dL and 2.1 mg/dL, respectively. He
denies any history of renal disease. Which one of the following options would be most
helpful in determining the etiology of this patient’s renal failure?
A. Abdominal ultrasound
B. CT abdomen
C. Urinary albumin
D. Urinary Calcium
E. No further work-up is necessary
71. A nursing assistant is tasked with cleaning a patient’s room following discharge. The
patient had diarrhea and was diagnosed with Clostridium difficile. Which disinfectant is
most effective for cleaning surfaces contaminated with C. difficile spores?
72. A 63-year-old man presents with a 2-day history of dull left-sided loin pain. He has a
known history of a left-sided renal calculus. On examination, he is pyrexial at 38.5° C, with
rigors. Urea 9.9 mmol/L, creatinine 155 μmol/L. A CT KUB confirms a large left pelvi-ureteric
junction calculus with left-sided hydronephrosis. What is the appropriate management?
73. A 59-year-old woman undergoes a laparotomy for suspected ovarian cancer; her family
asks if you would disclose the results to them so they can explain better to her. Which of
the following ethical principles should guide you to take your decision?
A. Autonomy
B. Beneficence
C. Paternalism
D. Confidentiality
E. Fidelity
74. A full-term newborn boy is delivered via normal spontaneous delivery. Within the first 2
hours after birth, he became tachypneic, with occasional apneas and two seizure episodes.
The infant is large for gestational age, and his weight is in the 95th percentile for his age.
There are no evident dysmorphic features, and the rest of the physical examination is
unremarkable. Serum studies demonstrate a blood glucose level of 30 mg/dL. Which of the
following conditions is most likely present in the mother?
A. Diabetes mellitus
B. Graves disease
C. Hepatic cirrhosis
D. Rheumatoid arthritis
E. Seizure disorder
75. A 27-year-old woman presented with a 5-year history of abdominal bloating, weight
gain, and constipation, as well as recent anal discomfort with no bleeding. The bloating
was eased by bowel opening, which occurred every 5–7 days. Stool consistency varied
from pellet-like to semi-formed. She strained at stool on every occasion. What is the most
likely finding on rectal examination?
A. A tear in the anal skin is normal with chronic straining, and unlikely to be a cause of
her pain
B. Contraction of the internal anal sphincter is normal on bearing down
C. Low resting anal pressure excludes an anal fissure
D. Rectal prolapse is possible despite her young age
E. The presence of stool in the rectum confirms the diagnosis of fecal impaction
76. A 32-year-old pregnant woman at 26 weeks of gestation presents to the hospital with
minimal painless bleeding following sexual intercourse. She is hemodynamically stable
and has had no such episodes in the past. Which of the following confirms the diagnosis of
placenta previa?
77. A 56-year-old woman with a long history of untreated hypertension is brought to the
emergency department because of a severe headache and confusion. The patient is
oriented to person, but not to time or place. Her blood pressure is 230/140 mm Hg, pulse is
86/min, and respirations are 18/min. Funduscopic examination reveals optic disk edema,
and a dipstick test shows protein in the urine. Which of the following is the most
appropriate pharmacotherapy?
A. Furosemide
B. Hydralazine
C. Nifedipine
D. Nitroglycerin
E. Sodium nitroprusside
78. A 36-year-old woman was brought to the emergency room with a sudden onset of
breathing difficulties. She has a notable history of smoking, raising suspicion of a
pulmonary embolism. What is the gold standard test for confirming this diagnosis?
A. ABG
B. Chest X-ray
C. D-Dimer
D. Pulmonary angiogram
E. Pulmonary function testing
79. A 30-year-old primigravida comes for her booking visit. Her sister had deep vein
thrombosis in her legs last year and suffered much pain and discomfort. She has heard
that pregnancy increases the risk for venous thrombosis and wants you to address her
concern. Which of the following is the most appropriate next step in patient care?
A. Warfarin
B. Test for thrombophilias
C. Low-dose aspirin
D. Heparin
E. Reassure
80. A man has the risks and potential benefits of his cholecystectomy explained to him.
Which ethical principle is best described here?
A. Gillick competence
B. Assent
C. Consent
D. Autonomy
E. Capacity
EMLE November 2024 Exams 21
81. A 6-day-old girl born at 32 weeks gestational age. She became lethargic and
hypothermic over the last 24 h. She is not tolerating her formula feeds, has 2-episodes of
bilious vomitus, and 3-times bloody diarrhea. On examination: there were abdominal
distension, visible bowel loops, abdominal wall erythema, and absent bowel sounds. Which
of the following is the most likely diagnosis?
A. Hirschsprung’s disease
B. Duodenal atresia
C. Esophageal atresia
D. Necrotizing enterocolitis
E. Meconium ileus
82. During an outbreak of norovirus in a hospital ward, several patients and staff members
become ill. The infection control team recommends strict adherence to protocols to contain
the outbreak. Which of the following is the MOST effective measure to prevent the spread
of norovirus?
83. A 50-year-old woman was diagnosed with perimenopausal bleeding. During her D&C
biopsy for that bleeding, the gynecologist suspected uterine perforation by the uterine
sound. Which of the following is the most appropriate next step in patient care?
85. A 70-year-old man complains of intermittent left iliac fossa, pain, anorexia, and anergia.
He has lost 2kg of weight. He has had Low-grade fevers for the last 3 nights. There is a
tender mass in the left lower quadrant. What is the most likely diagnosis?
A. Appendix mass
B. Colonic carcinoma
C. Lymphoma
D. Palpable bladder
E. Pelvic abscess
86. A 75-year-old woman presents with a two-month history of recurrent, brief episodes of
vision loss in her right eye. Her ECG is normal, and a carotid duplex scan reveals 30%
stenosis of the right internal carotid artery. Which of the following is the recommended
management for this patient?
87. A 72-year-old woman with 3 months of change in bowel habit including frequent
diarrhea and urgency but no blood in the stool. Her weight is stable and there are no
masses palpable. What is the single most appropriate first investigation?
88. A 40-year-old woman presents with a history of a slowly growing right-sided lump at the
base of the neck. On examination, there is a thyroid nodule with associated
lymphadenopathy. She has a recent history of adrenalectomy. What is the most probable
diagnosis of the thyroid mass?
A. Graves’ disease
B. Follicular adenoma
C. Medullary carcinoma
D. De Quervain’s thyroiditis
E. Papillary carcinoma
89. A 35-year-old firefighter is admitted following a house fire with third-degree burns
covering 50% of his body. His estimated weight is 80 kg. How much fluid should be
administered in the first 8 hours?
A. 5L
B. 6L
C. 8L
D. 14 L
E. 16 L
EMLE November 2024 Exams 23
90. A 55-year-old woman, who recently had been dieting with a weight loss of 20 lb,
presents with a small-bowel obstruction and pain, which radiates down the inside of her
thigh to the knee. She has no past history of abdominal surgery. Which of the following is
the likely diagnosis?
91. A 22-year-old woman presents to her GP for advice regarding the most appropriate
postnatal contraception. She had an uncomplicated vaginal delivery 6 weeks back at 40
weeks gestation. She is intermittently breastfeeding and bottle feeding her baby. She and
her partner are keen to space out childbearing by 2–3 years and requesting the most
reliable form of contraception. She admits to having difficulty remembering to take
contraceptive medication. Which of the following is the most effective contraceptive option
for her?
A. Lactational amenorrhea
B. Progestogen only implant
C. Condoms
D. Combined oral contraceptive pill
E. Combined contraceptive vaginal ring
92. A 25-year-old man comes to the clinic complaining of enlarged, rubbery, non-
erythematous, painless, non-tender cervical lymphadenopathy. He claims to having weight
loss, fever, and night sweats. Which of the following is the best initial diagnostic step in the
management of this patient?
93. A 76-year-old woman presented by fracture neck of left femur and treated by total hip
replacement. 4 days after surgery, the patient developed dyspnea and chest pain. On
examination, the temperature was 37.8, RR 32/min, pulse 105B/min, right lower limb edema.
Chest x-ray showed unremarkable findings. D-dimer was 1012μg/L (N:<250μg/L). What is
the most suspected diagnosis?
94. A 40-year-old surgeon visits the outpatient department with complaints of fever,
weakness, right upper quadrant abdominal pain, loss of appetite, and dark urine. He
reported experiencing a low-grade fever three days ago and mentioned a pinprick injury
while performing an appendectomy on a patient one month prior. Physical examination
shows deep jaundice and an enlarged, tender liver. An abdominal ultrasound reveals an
enlarged, bright liver. Laboratory tests indicate a serum bilirubin level of 15 mg/dL (normal
0.2–1.2 mg/dL), direct bilirubin at 8 mg/dL, serum alkaline phosphatase at 220 IU/L (normal
up to 130 IU/L), ALT at 360 (normal up to 45 U/L), AST at 260 (normal up to 40 U/L), and
urine bilirubin ++. What is the most likely underlying cause of his condition?
A. Hepatitis A
B. Hepatitis B
C. Alcoholic hepatitis
D. Hepatitis D
E. Hepatitis E
95. A 7-month-old boy was presented to the outpatient clinic with a low-grade fever,
wheezing, and a dry cough. Examination reveals a respiratory rate of 60 breaths/minute and
chest retractions. A chest X-ray shows hyperinflation and some infiltration. What is the
most likely diagnosis?
A. Croup
B. Epiglottitis
C. Bronchial asthma
D. Bronchiolitis
E. Pneumonia
96. A 34-year-old woman primigravida known to be a type II diabetic. During her vaginal
delivery, a shoulder dystocia was diagnosed and managed by overdue traction,
exaggerated lithotomy, and suprapubic pressure. Which of the following is the most likely
complication of her baby?
97. A 74-year-old female developed progressive constipation over the last 9 months. She
started to take oral laxatives which used to help but in the last few weeks she had to
defecate every few days by glycerin suppository. She noticed some abdominal distension
and occasionally passes fluidy stools and mucus. She reports some colic and bowel
sounds, both diminish if stools or flatus could be passed. What is the most probable cause
of her symptoms?
98. A 2-year-old girl is brought to the ER with confirmed rotavirus diarrhea. She is
tachycardic and lethargic with sunken eyes, poor skin turgor, and dry mucous membranes.
Which of the following is the most appropriate next step in management?
99. A 13-year-old girl is brought to the clinic complaining of shortness of breath and chest
discomfort during exercising. On examination, there is a double apical impulse and a harsh
midsystolic murmur loudest between the apex and the left sternal border. Her ECG shows
left ventricular hypertrophy (LVH) and widespread Q waves. Hypertrophic Obstructive
Cardiomyopathy (HOCM) was suspected. Which of the following is associated with
increased risk of sudden death?
100. A 12-month-old boy is brought to the well baby clinic to assess his developmental
milestone. Which of the following is appropriate for this infant's age?
A. Kicking a ball
B. Climbing up and down from furniture without assistance
C. Saying short sentences with two to four words
D. Building towers with four or more blocks
E. Standing unsupported
November 2024, 5th November
(Second Simulation Exam
Medicine)
EMLE November 2024 Exams 1
2. A neonate with Trisomy 21 develops bilious emesis on the first day of life. An abdominal
x-ray reveals a "double bubble sign." What is the most likely diagnosis?
A. Duodenal atresia
B. Gastric volvulus.
C. Annular pancreas.
D. Hirschsprung disease.
E. Pyloric stenosis
A. Alveolar proteinosis
B. Aspiration pneumonia
C. Chronic bronchitis
D. Emphysema
E. Vasculitis
4. A 3-year-old boy with fair hair color and bad smell of urine presented with intellectual
disability, he has an affected older sister with inborn error of metabolism. Which of the
following is a possible diagnosis?
A. Galactosemia
B. Tyrosinemia
C. Phenylketonuria
D. Mucopolysaccharidosis
E. Gaucher disease
2 EMLE November 2024 Exams
5. A pregnant woman with fibroid uterus develops acute pain in abdomen with low-grade
fever and mild leukocytosis at 28 weeks, which of the following is the most likely
diagnosis?
A. Preterm labor
B. Torsions of fibroid
C. Red degeneration of fibroid
D. Infection in fibroid
E. None of the above
6. A 55-year-old man presents to the emergency department with increasing itching and
upper abdominal discomfort. His wife has noticed that he is looking ‘yellow’. On
examination, there is a non-tender mass in the right upper quadrant. The patient has a
history of ulcerative colitis, which is currently in remission, which of the following is the
most likely diagnosis?
A. Cholangiocarcinoma
B. Gallstones
C. Hemolysis
D. Hepatitis
E. Pancreatic carcinoma
7. A patient at 34 weeks’ gestation develops marked pruritus especially on her palms and
soles, and mildly elevated liver function tests and elevated bile acids. Under which of the
following conditions do the pruritus and jaundice in this patient likely to recur?
A. Menopause
B. After discontinuation of breast-feeding
C. Poor diet
D. With another pregnancy
E. With the use of antihypertensive medication
8. A 35-year-old male patient undergoes an elective thyroid surgery. The patient is not
diabetic nor suffering from any systemic illness. How can you describe his wound?
A. Class I, Clean
B. Class II, Clean/contaminated
C. Class III, Contaminated
D. Class IV, Dirty
E. Wound to heal by secondary intention
9. A 77-year-old woman presents with vulvar itching and color changes in perineum. Which
of the following is the commonest white lesion of the vulva?
10. A 36-year-old male complaining of drops of fresh blood at the end of defecation, on and
off, for 6 months with no pain. The bleeding stops spontaneously after 5 to 10 minutes.
These attacks last for few days and recur few weeks later. The stools are well formed and
not mixed with blood. No passage of mucus. Lately the patient felt soft tissue protrusion
per anum when washing after defecation and cannot push it inside. Concerning this
condition, which of the following statement is true?
11. A 52-year-old man who developed hoarseness of voice following thyroid surgery 16
weeks ago and showing no improvement. Which anatomical site is most likely affected?
12. A healthy full term baby boy is born to mother who was confirmed to have acute
Hepatitis B infection during this pregnancy. Which is the most appropriate preventative
intervention to the baby?
13. A 42-year-old obese mother of 5 children comes with severe upper abdominal pain
mainly in the right side and referring to the right shoulder with a temperature 37.8°C. What
is the most probable diagnosis?
A. Biliary colic
B. Cholecystitis
C. Ectopic pregnancy
D. Hepatitis
E. Ovarian torsion
14. A 4-year-old boy brought to the emergency room with flushed skin, after he received an
intramuscular injection of antibiotic. Which of the following is a clinical manifestation of
anaphylaxis?
A. Cough
B. Hypotension
C. Hypertension
D. Cold skin
E. Sore throat
4 EMLE November 2024 Exams
15. A 67-year-old man is complaining of easy fatigability. A routine complete blood count
(CBC) reveals a platelet count of 800,000/ml, and the hemoglobin and WBC counts are
normal. He reports no other symptoms, and his clinical examination is normal. Which of the
following characteristics is most likely to be helpful in differentiating essential (primary)
from reactive (secondary) thrombocytosis?
16. A 34-year-old man presents to the emergency department with a 6-hour history of
abdominal pain and vomiting. On examination, the abdomen been having episodes of
cramping right-sided abdominal pain with intermittent episodes of blood in his stools.
Which of the following complications has resulted in his presentation today?
A. Abscess formation
B. Fistula formation
C. Primary sclerosing cholangitis
D. Stricture formation
E. Toxic megacolon
17. A 20-year-old woman comes to your clinic with irregular cycles since menarche and
mild hirsutism. She is not interested in pregnancy or contraception. Her serum TSH,
prolactin, and dehydroepiandrosterone sulfate (DHEAS) levels are normal, with a slightly
elevated serum testosterone level of 80 ng/dl. Which of the following is the most
appropriate next step for this patient?
18. An 8-month-old boy presents to the emergency room with jerky movement of all limbs.
He was a full-term baby, with no neonatal problems. He is exclusively breast fed. He is
afebrile with height and weight at 50th percentile. What is the next investigation to be
carried out?
A. Electromyography
B. Lumbar puncture
C. Nerve conduction velocity
D. Serum calcium
E. Serum sodium
EMLE November 2024 Exams 5
20. A 32-year-old primigravida at 39 weeks of gestational age has a blood pressure reading
of 150/100mm Hg obtained during a routine visit. Her baseline blood pressure during the
pregnancy was 120/70mmHg. The patient denies any headache, visual changes, nausea,
vomiting, or abdominal pain. Her repeat BP is 150/90 mmHg, and urinalysis is negative for
protein. Which of the following is the most likely diagnosis?
A. Preeclampsia
B. Chronic hypertension with superimposed preeclampsia
C. Eclampsia
D. Gestational hypertension
E. None of the above
21. A 3-month-old infant is found to have a continuous murmur all over the precordium
during her routine well baby checkup. The infant is 6 kg in weight and appears well. All
peripheral pulses are present and easily palpable. Oxygen saturation is 96%. What is the
most likely finding on echocardiography?
22. A 22-year-old man presented with cough with expectoration and night fever of one
month duration. The patient has therapeutic history of 2 courses of potent antibiotics
without improvement. Clinical examination revealed: Temp. 37.6°C, pulse 95 b/min. regular.
Bl. pressure 110/70, respiratory rate 22 breath/min. Chest examination showed bilateral
infraclavicular crepitation, which of the following is the recommended investigation to
prove the diagnosis?
A. X-ray chest
B. Ventilation perfusion scan
C. Pulmonary function tests
D. Bronchoscopy
E. Sputum for acid fast bacilli
6 EMLE November 2024 Exams
23. A 33-year-old woman developed bilateral loin pain, frank hematuria and edema in feet.
Her symptoms had started 24 hours after developing a sore throat, her blood pressure
was120/80 mmHg. Urine analysis was positive for blood (4+) and protein (2+). Which of the
following is the most likely diagnosis?
A. IgA nephropathy
B. Microscopic polyangiitis
C. Nephrolithiasis
D. Post-streptococcal glomerulonephritis
E. Septicemia
24. A 52-year-old man comes to the office because of a 2-day history of sore throat, cough
productive of a small amount of whitish sputum, nasal discharge, and body aches. He has
type 2 diabetes mellitus on metformin and hypertension treated with ACE inhibitors. He
smokes occasionally. His temperature is 38.0°C, pulse is 85/min, respirations are 22/min,
and blood pressure is 140/90 mm Hg. Pulse oximetry on room air shows an oxygen
saturation of 98%. Physical examination shows no cyanosis. The throat is mildly
congested. Auscultation of the lungs discloses scattered wheezes. Air entry is equal
bilaterally. Heart sounds are normal. Which of the following is the most likely diagnosis?
A. Asthma
B. Bronchiolitis
C. Bronchitis
D. Laryngitis
E. Pneumonia
26. A 25-day old infant diagnosed as Down syndrome was found to have a murmur. Which
of the following cardiac lesions is most likely in this baby?
A. Common AV Canal
B. VSD
C. ASD
D. D-TGA
E. PFO
EMLE November 2024 Exams 7
A. Pap smear
B. Punch biopsy
C. Colposcopy
D. Endocervical curettage
E. None of the above
28. A 28-year-old multiparous woman transfers her care to you for an annual follow up and
contraceptive counseling. She has used combined oral contraceptive pills COCs 5 years
ago. She would like to restart contraception now. She has no ongoing medical illnesses
and takes no medications. Physical examination reveals a 12-14-week, irregular uterus
suggestive of uterine leiomyomata. Based on these findings, the most appropriate
contraceptive method for this patient would be which of the following?
29. A 55-year-old woman presented with intermittent attacks of moderate vaginal bleeding
since two months. She stopped menstruation two years ago, which of the following is the
appropriate diagnostic tool for her?
A. Transvaginal ultrasound
B. CA-125
C. Pap smear
D. Fractional curettage
E. Pregnancy test
30. A 23-year-old woman without prenatal care (gravida 1, para 0) in the third trimester of
pregnancy arrives in the emergency department. She has presented because of headache
and visual change. While being examined, she had a convulsion. What is the most
important step that should be done before giving magnesium sulfate bolus?
31. A 42-year-old, Para 4 pregnant at 38+4 weeks consulted her doctor about recurrent,
painless mild attacks of vaginal bleeding during last week. She is clinically free but
ultrasound scan showed a posterior placenta previa marginalis. What is the appropriate
management?
32. A 19-year-old female with moon face and a hump on the upper back. On examination:
pulse 90 bpm regular, BP was 150/90 mmHg and abdominal striae. What is the first lab test
change found in this patient?
33. A 31-year-old G2P2 has a Cesarean Section for severe preeclampsia (PET) and 3 days
post operatively, she complains of chest pain, shortness of breath and central cyanosis.
Which one of the following is the most likely diagnosis?
A. Carcinoid tumor
B. Small cell lung cancer
C. Cardiac myxoma
D. Adrenal carcinoma
E. Multiple pregnancy
EMLE November 2024 Exams 9
35. A five-year-old boy presents to the outpatient clinic with generalized edema which is
more apparent in the morning. Respiratory rate 18/min, Heart rate 90/ min, BP 80/45.
Laboratory investigations show serum albumin= 1.4 g/dL (3.5-5.5 gm/dl), cholesterol = 360
mg/dL (150-199 mg/dl), urinalysis revealed +++ proteins and RBC 1-2/HPF. Serum creatinine
is normal. The physician requests a kidney biopsy. What is the most probable Pathological
diagnoses?
36. A 67-year-old man is complaining of easy fatigability. A routine complete blood count
(CBC) reveals a platelet count of 800,000/ml, and the hemoglobin and WBC counts are
normal. He reports no other symptoms, and his clinical examination is normal. Which of the
following characteristics is most likely to be helpful in differentiating essential (primary)
from reactive (secondary) thrombocytosis?
37. A one-year-old boy is brought to the clinic by his mother for routine well child
examination. He is exclusively breastfed till now. The mother says she tried to introduce
cereals, but he refused. On examination the child is pale with no organomegaly. The GP
requests a blood picture which shows a Hg 8.7 gm/dl (N: 14-17 gm/dl) and HCT 24% (N:31-
41%). What is the most likely diagnosis?
A. Spherocytosis
B. Iron deficiency anemia
C. Pernicious anemia
D. Sideroblastic anemia
E. Thalassemia
38. A 30-year-old man presents with a left-sided dragging pain and swelling in the left hemi-
scrotum. On examination, he has a left-sided swelling that is palpated separately from the
testes and feels 'like a bag of worms'. The swelling disappears on lying supine. Which of
the following is the most likely diagnosis?
A. Hydrocele
B. Indirect inguinal hernia
C. Testicular tumor
D. Epididymal cyst
E. Varicocele
10 EMLE November 2024 Exams
39. A pregnant woman not previously known to be diabetic, who is at 26 weeks' gestation,
had a routine 50-g oral Glucose Tolerance Test (OGTT) with a 1-hour blood glucose value
of 144 mg/dl (Normal value <130 mg/dl). Accordingly, next day a follow up of 100-g, 3-hour
oral Glucose Tolerance Test (OGTT) revealed plasma values of fasting blood sugar of 102;
1 hour, 180; 2 hours, 162; and 3 hours, 144 mg/dl (Normal values: fasting <95mg/dl, 1 hour=
180 mg/dl, 2 hours 155 mg/dl, 3 hour= 140 mg/dl). Which of the following should be your
next management?
40. A female child walks well and runs stiffly. She kisses her parents and helps with
housework. She says 10 words but cannot put 3 words together. She uses words for her
needs. She feeds herself with a spoon. She can build a tower with 4 cubes. What is her
most likely age in months?
A. 12
B. 15
C. 18
D. 24
E. 30
41. A 61-year-old man, with an 8-year history of hepatitis C infection and well-documented
cirrhosis and portal hypertension, presents with a large hematoma on his thigh. On
preoperative screening, his prothrombin time is noted to be 17.4 seconds (N: 11-13.5
seconds). Transfusion of which of the following is the most appropriate next step in
management of this patient prior to his procedure?
A. Cryoprecipitate
B. Fresh frozen plasma
C. Packed red blood cells
D. Platelets
E. Whole blood
42. A 40-year-old man is being investigated for an enlarging painless lump in the right
testicle. Blood results show a significantly raised alpha fetoprotein and a normal beta-hCG.
Which of the following testicular tumors is most likely to be the cause of his symptoms?
A. Choriocarcinoma
B. Leydig cell tumor
C. Seminoma
D. Testicular lymphoma
E. Yolk sac carcinoma
EMLE November 2024 Exams 11
43. You are called to see an 85-year-old female patient as the nursing staff is concerned
that the patient has not passed stool for 4 days. The patient has been admitted after family
members became increasingly concerned regarding her general deterioration in health and
level of function. She is orientated but frail and complains of increasing abdominal
discomfort. On examination bowel sounds are increased. The abdomen is distended with
generalized tenderness, but no rebound or guarding. There is a firm palpable mass in the
left iliac fossa. Digital rectal examination shows an empty rectum. What diagnosis must be
excluded?
A. Simple constipation
B. Paralytic ileus
C. Sigmoid volvulus
D. Peritonitis secondary to diverticular disease
E. Neoplasia
44. A 14-year-old boy notices tingling about his ankles 2 weeks after an upper respiratory
tract infection. Within two days, he has weakness in dorsiflexion of both feet, and within
one week he develops problems with walking. He has no loss of bladder or bowel control.
His weakness progress rapidly over the ensuing week and necessitates his being placed
on a ventilator to support his breathing. He is quadriplegic, but retains control of eye
movements. Cerebrospinal fluid studies reveal a protein content of greater than 1 g/dL with
a normal white cell count. There are no blood cells in the CSF. Which of the following is the
condition that best fits this clinical scenario?
45. A 75-year-old man presents to the Emergency Department with sudden onset frank
hematuria. Five days earlier he required catheterization following an episode of
postoperative urinary retention after an elective inguinal hernia repair. The catheter was
removed on discharge and he has passed urine freely since. On examination, the urine is
frank hematuria with evidence of blood clots. What is the most likely diagnosis?
46. A 14-year-old boy notices tingling about his ankles 2 weeks after an upper respiratory
tract infection. Within two days, he has weakness in dorsiflexion of both feet, and within
one week he develops problems with walking. He has no loss of bladder or bowel control.
His weakness progress rapidly over the ensuing week and necessitates his being placed
on a ventilator to support his breathing. He is quadriplegic, but retains control of eye
movements. Cerebrospinal fluid studies reveal a protein content of greater than 1 g/dL with
a normal white cell count. There are no blood cells in the CSF. Which of the following is the
condition that best fits this clinical scenario?
47. A female patient, 75-year-old presented with epigastric mass & weight loss with change
in bowel habit towards constipation, the patient passed stool today in the morning.
Colonoscopy was done early and revealed a nearly obstructing mass in the transverse
colon and biopsy showed adenocarcinoma grade III. What is the surgical option?
48. A 31-year-old man is brought to the emergency room following a motor car accident in
which his chest struck the steering wheel. Examination reveals stable vital signs, but the
patient exhibits multiple palpable rib fractures and paradoxical movement of the right side
of the chest. Chest X-ray shows no evidence of pneumo-or hemothorax but a large
pulmonary contusion is developing. What is the proper management for this case?
49. A 5-year-old boy has an itching sensation around his anus, is worse at night which
interrupt him from sleeping well. Which investigation can most likely help confirm the
diagnosis?
A. Rectal swab
B. Direct stool smear method
C. Peri-anal swab examination
D. Peri-anal swab culturing
E. Stool analysis by concentration
EMLE November 2024 Exams 13
50. A-60-year-old diabetic man presented to the ER by central chest pain at rest of 4 hours
duration. Clinical examination revealed pulse 100 b/min. regular, Bl. pressure 100/60. Chest
and cardiac examination are unremarkable. ECG showed regular sinus rhythm without ST
segment elevation. Which of the following is the most suitable investigation to reveal the
diagnosis?
51. A 20-year-old woman has just delivered a full-term baby in the Emergency Department.
The baby has been unresponsive and apneic. CPR has been commenced. Which one of the
following is the most appropriate chest compression to inflation ratio?
A. 3:1
B. 5:1
C. 3:2
D. 5:3
E. 6:2
52. Reviewing the chest X-ray of a preterm newborn, you observe diffuse, fine,
reticulogranular densities, which provide a ground-glass appearance. On the basis of these
radiographic findings, which of the following conditions should you suspect?
53. An 18-years old healthy woman came to your clinic with a recent purified protein
derivatives tuberculin skin test that is positive after routine examination for employment.
Clinical examination was irrelevant with no signs of disease. Chest examination was free. A
chest radiograph was done and is negative. What is the most appropriate treatment?
54. A 30-year-old Para 3 asked her doctor about her chances of getting a boy after having 3
girls. What is the proper explanation of her fetal sex determinant?
55. A 22 years old primigravid woman with unremarkable previous ANC is having on a
routine urine analysis 1-2 pus cells/HPF, but the bacterial colony count was more than
100,000/mi which of the following comments should be delivered to the lady?
56. A 6-month-old healthy infant presents to the immunization room in a primary health
care center for vaccination. Which vaccines should be given to him at this age, according
to Egyptian vaccination schedule?
A. BCG vaccine, Salk vaccine, Oral polio vaccine and pentavalent vaccine
B. MMR, pentavalent vaccine and Oral Polio vaccine
C. Oral Polio vaccine and Pentavalent vaccine
D. Pentovalent vaccine, rubella vaccine and Vitamin A capsule
E. Rubella vaccine, Hemophilus influenza b vaccine, pentavalent vaccine
57. A 3-year-old boy presents with anorexia, malaise, fever 37.7oC, and maculopapular rash
that started on the face and spread to the body for the past 3 days. On examination there is
retro auricular and posterior occipital lymphadenopathy. CBC results show: WBC is
4500/ml and direct platelet count is 11000/mm. What is the most probable diagnosis?
A. Kawasaki disease
B. Measles
C. Roseola
D. Rubella
E. Scarlet fever
58. A 34-year-old man is extracted from an automobile after a motor vehicle collision. The
patient has an obvious deformity of his right thigh consistent with a femur fracture. Upon
closer examination of the right thigh, there is bone visible through an open wound. Which
of the following is the appropriate management of his open femur fracture?
59. A 60-year-old man with history of T2 DM and chronic obstructive pulmonary disease
(COPD) is hospitalized for COPD exacerbation. His medications include insulin glargine 40
u/d, metformin 1000 mg BID, and inhaled fluticasone plus salmeterol He reports that a
recent Ale level was 7.0% (normal, 3.8-5.6). On examination, he is in mild respiratory
distress. Temperature is 37.9nC, blood pressure is 146/90 mm Hg, heart rate is 102 b/m,
respiratory rate is 22 b/m, and oxygen saturation is 90% while he receives 4 liters of
supplemental oxygen by nasal cannula. The patient is started on levofloxacin, oral
prednisone 60 mg daily, and nebulized albuterol plus ipratropium. His home insulin
glargine dose is continued, and the metformin is discontinued. On the second hospital day,
several random glucose levels are between 200 and 300 mg/dl (normal <140). Which one of
the following strategies is best for managing this patient's hyperglycemia?
A. Divide the insulin glargine dose in half and administer the drug twice daily
B. Add prandial insulin
C. Replace prednisone with an equivalent dose of dexamethasone
D. Start a long-acting sulfonylurea as monotherapy
E. Increase the insulin glargine dose
60. A diabetic mother gives birth to a full-term neonate with a birth weight of 5 kgs. On
examination the baby is active and pink with good neonatal reflexes and mild jitteriness.
Heart rate & respiratory rate are within normal. Which biochemical test is essential during
the initial neonatal examination?
A. Blood glucose
B. Blood hemoglobin
C. Serum calcium
D. Serum magnesium
E. Serum sodium
61. A 63-year-old man, with an 8-year history of hepatitis C infection and well-documented
cirrhosis and portal hypertension, presents with a large hematoma on his thigh. On
preoperative screening, his prothrombin time is noted to be 17.4 seconds (N: 11-13.5
seconds). Transfusion of which of the following is the most appropriate next step in
management of this patient prior to his procedure?
A. Cryoprecipitate
B. Fresh frozen plasma
C. Packed red blood cells
D. Platelets
E. Whole blood
62. A 30-year-old pregnant mother has a routine third trimester ultrasound that shows
bilateral fetal hydronephrosis and hydroureter and decreased amniotic fluid. What is the
most probably diagnosis?
63. A 45-year-old woman with ulcerative colitis is admitted with a history of jaundice,
pruritus, and intermittent abdominal pain. Examination shows hepatosplenomegaly and
mild ascites. Blood tests confirm an obstructive jaundice, and mitochondrial antibodies are
not detected, which of the following is the most likely diagnosis?
64. A 6-year-old boy is diagnosed with a coagulation abnormality. He was bleeding heavily
after the tonsillectomy. Laboratory results reveal decreased factor VIII C, increased
bleeding time, normal PT, and increased PTT. Which one of the following is the most likely
diagnosis?
A. Hemophilia A
B. Hemophilic B
C. Hemophilia C
D. Carrier hemophilia
E. Von-Willebrand disease
65. A newly married woman mentions to you, that she had a positive pregnancy test and
wonders if you can calculate her expected date of delivery (EDD). The first day of her last
menstrual period (LMP) was June 30. Which of the following will be her expected date of
delivery?
A. 44986
B. 39173
C. 46813
D. 45017
E. 39142
66. A 48-year-old, Para 5 widow consulted her doctor about her abnormal menstrual
pattern. Over the last year she menstruated heavily every 2-3 months, her physical
examination was unremarkable. Transvaginal pelvic ultrasound revealed a mildly enlarged
uterus with endometrial thickness 17mm. Which one of the following is the appropriate
next step of management?
67. A 40-year-old woman is brought to ER 3 hours following the abrupt onset of severe
deep epigastric pain, nausea, and vomiting. The pain is steady and radiates to the back.
The patient is agitated and has cool, clammy skin. Her temperature is 38.5 C, BP is 100/70
mm Hg, pulse is 110/bpm, and RR are 22/min, abdominal exam, reveals tenderness in the
upper abdomen, without guarding. A plain x-ray film shows an air-filled intestinal loop in
the left upper quadrant. Abdominal sonography shows multiple gall stones without
dilatation of bile ducts. Laboratory investigations show: fasting glucose 150 mg/dL, ALT
90u/l (n40), AST 80 u/l (n40), Amylase 620 UIL (N 30-110), Lipase 510 u/l (n10-140). Which of
the following is the most likely diagnosis?
A. Acute cholecystitis
B. Acute hepatitis
C. Acute pancreatitis
D. Bowel perforation
E. Mesenteric ischemia
68. A 9-month child is brought to the emergency room with on irreducible firm swelling
which descended into the left groin when the child has been crying. On examination, both
testicles are palpable in the scrotum. What is the appropriate management strategy?
A. Elective herniotomy
B. Elective herniotomy + orchidopexy
C. Emergency herniotomy
D. Emergency herniotomy + orchidopexy
E. Reassurance
69. A 3-month-old infant is brought to the clinic by his mother with clear nasal discharge
and mild cough. The general practitioner explains he has a mild viral infection. Two days
later the infant presents to the Emergency Room with cough, difficult breathing and
wheezes. Temp was 38° C, Respiratory rate 28/min, Heart rate 110/min. Examination reveals
intercostal retractions, diminished air entry on both lungs. What is the most likely
diagnosis?
A. Bronchial asthma
B. Bronchiolitis
C. Epiglottitis
D. Foreign body aspiration
E. Pneumonia
A. Gardner’s syndrome
B. Hereditary juvenile polyposis
C. Lynch syndrome
D. Peutz–Jeghers syndrome
E. Sipple’s syndrome
18 EMLE November 2024 Exams
71. A 25-year-old PG came for her first antenatal visit at 6 weeks gestation. She was
clinically free with satisfactory laboratory tests. What would be the recommended ROUTINE
supplement(s) as in all pregnancies?
72. A 30-year-old woman presents with neglected labor. Urine output chart was estimated
to diagnose suspected dehydration. At which of the following level of urine volume oliguria
is diagnosed?
73. A 60-year-old heavy smoker man presented with hemoptysis for one month. CT showed
bronchogenic carcinoma with liver metastases. The diagnosis was confirmed by
bronchoscopic biopsy. To whom should you explain the situation and prognosis?
A. Brother
B. Daughter
C. Son
D. Patient himself
E. Wife
74. A 25-year-old woman is brought to the Emergency Department following a road traffic
car accident. On arrival her blood pressure is 110/50mmHg, heart rate 92/min and
respiratory rate 28/min. On examination there is decreased air entry on the right side as
well as dullness on percussion and fracture of 2 ribs. Which one of the following is
immediate treatment of the patient’s condition?
A. Analgesia-sedatives
B. CPAP
C. Endotracheal intubation and ventilation
D. Insertion of chest drain underwater seal
E. Strapping the complete chest
75. A 52-year-old man who developed hoarseness of voice following thyroid surgery 16
weeks ago and showing no improvement. Which anatomical site is most likely affected?
76. A 55-year-old man comes to the office because of a one-day history of sudden onset of
severe pain in his low back that radiates down his left leg, and a pins and needles feeling in
his left heel. The pain increases if he remains in a static position for longer than 10
minutes. He has had difficulty sleeping due to pain. He has had 3 episodes of moderate low
back pain that did not radiate during the past 4 months Days//weeks//months//years. During
past episodes, nonsteroidal anti-inflammatory drugs and muscle relaxants provided
complete relief of his symptoms. During this current episode he has no other history of
serious illness and takes no routine medication. His temperature is 370℃, pulse is 85/min
and regular, respirations are 16/min, and blood pressure is l40/100 mm Hg.
Cardiopulmonary and abdominal examinations show no abnormalities. Bladder and bowel
functions are normal. Sensation in the groin is intact. Muscle strength is normal in the
lower extremities. Deep tendon reflexes are normal bilaterally. Flexion of the left lower
extremity is limited to 30 degrees due to pain. Which of the following is most likely cause of
this patient’s symptoms diagnosis?
A. Herniated disc
B. Myocitis
C. Spinal stenosis
D. Spinal tumor
E. Spondylolisthesis
A. Gastric carcinoma
B. Esophageal carcinoma
C. Opiate analgesia
D. Pancreatitis
E. Gastritis
A. B-Blockers
B. Calcium channel blockers
C. Digitalis
D. ACE-I
E. D.C shock
20 EMLE November 2024 Exams
79. A 6- year- old boy with Down syndrome is brought by his mother to the clinic with
prolonged fever, weight loss and pallor. which of the following is the most appropriate
initial investigation?
80. A 27-year-old man complains at perianal itching bleeding, discharge, pain and lumps for
a few months. On examination he has pinkish-white swelling outside and inside the anal
canal partially obscuring the anal orifice. What is the appropriate diagnosis?
A. Anal fissure
B. Fistula in-ano
C. Hemorrhoids
D. Hidradenitis suppurativa
E. Sebaceous cyst
82. A 28-year-old infertile woman is known to have endometriosis. She asked you to explain
what is ment by conservative surgical treatment. Which of the following is not to mentioned
to her?
83. A 45-year-old woman presents with a one-week history of a painful left ankle. On clinical
examination, she looked flushed and had a temperature of 37.5 C°. The joint was red, hot,
and had reduced movement as a result of pain. The rest of her joints were relatively
asymptomatic. Nine years previously, she had been found to have rheumatoid arthritis, for
which she was taking Methotrexate, and her disease had been recently documented as in
remission. What is the most likely cause of her painful ankle?
A. Gout
B. Flare of her rheumatoid arthritis
C. Septic arthritis
D. Pseudo gout
E. Reactive arthritis
84. A 27-year-old man presents with pain in his left testis that has been present for the past
2 days. The pain has been getting gradually worse and there is now also noticeable
swelling of the testis. On examination he is febrile with a temperature of 38.3°C and the
scrotum appears red and edematous on the affected side. The testis is tender on palpation.
Which one of the following is the most likely diagnosis?
A. Scrotal hematoma
B. Testicular torsion
C. Epididymo-orchitis
D. Torsion of hydatid of Morgagni
E. Irreducible inguinoscrotal hernia
85. A 24-year-old man presents with a swollen right leg following a flight from Saudi Arabia
to the UK. He has no past medical history or relevant family history. On examination there
is a one cm difference in calf circumference with minimal pitting edema. There is no
tenderness to palpation over the deep veins and no evidence of collateral blood flow. His
clinical risk of deep vein thrombosis is low. Which of the following blood tests is most
likely to exclude thrombosis in this patient?
A. C-reactive protein
B. D-dimer
C. Fibrinogen
D. INR
E. Platelet count
86. A 35-year-old P0+1, aborted 5 months back at 17 weeks of a gestation. She has not got
her periods yet. Urine pregnancy test is negative. Estrogen progesterone withdrawal test is
negative. Which one of the following is the most likely diagnosis?
A. Pituitary failure
B. Ovarian failure
C. Anovulation
D. Asherman syndrome
E. None of the above
22 EMLE November 2024 Exams
88. A woman in labor develops bleeding at 5-cm dilation with fetal distress. You perform a
cesarean section and find a Couvelaire uterus. Which of the following causes a Couvelaire
uterus?
89. A 26-year-old man comes to the physician because of sore throat, fever, and malaise for
1 week and a diffuse skin rash for 1 day. The skin rash developed after the patient took
ampicillin. Examination reveals pharyngitis and tonsillitis, cervical lymphadenopathy, and
splenomegaly. Laboratory studies showed: • Hematocrit 40%, • Leukocytes count
4300/mm3 • Segmented neutrophils 45% • Lymphocytes 40% • Platelets count 76,000/mm3 •
Alanine aminotransferase 80 U/L (10 to 40 U/l) • Aspartate aminotransferase 70 U/L (10 to 40
U/l) • Total bilirubin 1.2 mg/dL • Peripheral blood smear showed numerous atypical large
lymphocytes with vacuolated cytoplasm. A heterophil antibody test is positive. Which of
the following is the most likely diagnosis?
90. An 8-year-old boy drought to the emergency room after accidentally touching a hot iron
with his forearm. On examination, the burned area has weeping blisters containing clear
fluid and severely tender. Which of the following is the burn depth?
A. First degree
B. Fourth degree
C. Scald
D. Second degree
E. Third degree
EMLE November 2024 Exams 23
91. A 26-year-old lady P2+0 presented with secondary amenorrhea of three years duration.
FSH and LH were found to be high. What is the most likely diagnosis?
A. Sheehan syndrome
B. Asherman syndrome
C. Premature ovarian failure
D. Imperforated hymen
E. Pituitary adenoma
92. A 5-year-old male presents with confirmed rotavirus diarrhea. He is tachycardic and
lethargic with sunken eyes, poor skin turgor, and dry mucous membranes. Which of the
following is the most appropriate next step in management?
93. A 45-year-old woman with ulcerative colitis is admitted with a history of jaundice,
pruritus, and intermittent abdominal pain. Examination shows hepatosplenomegaly and
mild ascites. Blood tests confirm an obstructive jaundice, and mitochondrial antibodies are
not detected, which of the following is the most likely diagnosis?
95. A 16-month-old boy brought to the Emergency Room with o 3-day history of fever and
cough. His temperature was 38.5°C. On examination, he is toxic. HR 140/ minute, RR
52/minute, and SPO2 is 82% on room air. There are markedly decreased breath sounds
over the right lung. A chest x-ray reveals an opacified right lung with central mediastinum.
The CBC shows a white count of 28,000/cmm (normal- 4,000-13,000/cmm) with evidence of
shift to the left. Which one of the following is the initial management step?
96. A 65-year-old woman with a past medical history of hypertension and stable angina
presents to the GP practice complaining of episodes of severe central abdominal pain.
They occur around half an hour after meals and can last up to an hour. She is becoming
reluctant to eat because of the pain and as a result she has lost nearly a stone in weight
over the past 2 months. Examination is unremarkable. Which of the following of these
investigations would be the most informative?
97. A 31-year-old women presents to you with recurrent abdominal pain and frequent
bloody diarrhea. You suspect inflammatory bowel disease and arrange for a colonoscopy
with biopsy. Which one of the following features on biopsy would suggest ulcerative colitis
over a diagnosis of Crohn’s disease?
A. Skip lesions
B. Rosethorn ulceration
C. Presence of granulomas
D. Transmural (full thickness) involvement
E. Presence of crypt abscess
98. A 4-week-old boy has repeated attacks of vomiting since birth that occur soon after
feeding, irrespective of the amount of milk he takes. He is otherwise well. Birth weight was
3.5 kg and on examination his weight is 4.4kg and no other abnormality is detected. What is
the most likely diagnosis?
A. Mitral stenosis
B. Mitral regurgitation
C. Aortic stenosis
D. Aortic regurgitation
E. Coarctation of the aorta
100. A 26-year-old para 1 had a missed period for 8 days 2 months after IUCD removal.
Serum B-hCG level was 550 u/L, raised to be 875 u/L after 48 hours with no evidence of
intra- uterine sac by transvaginal ultrasound. What is the most likely diagnosis?